Exam 3 Pharmacology Study Questions

¡Supera tus tareas y exámenes ahora con Quizwiz!

Ethnic groups differ in the their risk for and presentation of diabetes. Hispanics :

( Both A and B)Have a high incidence of obesity , elevated triglycerides, ad hypertension/ Do best with drugs that foster weight loss, such as metformin

Screening criteria for children who meet the following criteria should begin at age 10 and occur every 3 years thereafter:

(ALL of the Above )BMI above 85th percentile, Fmaily history of diabetes in first and second degree relative, hypertension based on criteria for children.

Infants with congenital hypothyroidism are treated with: 1. Levothyroxine 2. Liothyronine 3. Liotrix 4. Methimazole

1

Treatment of a patient with hypothyroidism and cardiovascular disease consists of: 1. Levothyroxine 2. Liothyronine 3. Liotrix 4. Methimazole

1

Dosing for caster oil

1-5mL <2 5-15mL (2-12) 15-60mL (Adult)

Migraines in pregnancy may be safely treated with: 1. Acetaminophen with codeine (Tylenol #3) 2. Sumatriptan (Imitrex) 3. Ergotamine tablets (Ergostat) 4. Dihydroergotamine (DHE)

1. Acetaminophen with codeine (Tylenol #3)

Migraines in pregnancy may be safely treated with: 1. Acetaminophen with codeine (Tylenol #3) 2. Sumatriptan (Imitrex) 3. Ergotamine tablets (Ergostat) 4. Dihydroergotamine (DHE)

1. Acetaminophen with codeine (Tylenol #3)

Henry is 82 years old and takes two aspirin every morning to treat the arthritis pain in his back. He states the aspirin helps him to "get going" each day. Lately he has had some heartburn from the aspirin. After ruling out an acute GI bleed, what would be an appropriate course of treatment for Henry? 1. Add an H2 blocker such as ranitidine to his therapy. 2. Discontinue the aspirin and switch him to Vicodin for the pain. 3. Decrease the aspirin dose to one tablet daily. 4. Have Henry take an antacid 15 minutes before taking the aspirin each day.

1. Add an H2 blocker such as ranitidine to his therapy.

Jamison has been prescribed citalopram (Celexa) to treat his depression. Education regarding how quickly selective serotonin reuptake inhibitor (SSRI) antidepressants work would be: 1. Appetite and concentration improve in the first 1 to 2 weeks. 2. Sleep should improve almost immediately upon starting citalopram. 3. Full response to the SSRI may take 2 to 4 months after he reaches the full therapeutic dose. 4. His dysphoric mood will improve in 1 to 2 weeks.

1. Appetite and concentration improve in the first 1 to 2 weeks.

Type 1 diabetes results from autoimmune destruction of the beta cells. Eighty-five to 90% of type 1 diabetics have: 1. Autoantibodies to two tyrosine phosphatases 2. Mutation of the hepatic transcription factor on chromosome 12 3. A defective glucokinase molecule due to a defective gene on chromosome 7p 4. Mutation of the insulin promoter factor

1. Autoantibodies to two tyrosine phosphatases

When prescribing Adderall (amphetamine and dextroamphetamine) to adults with ADHD the nurse practitioner will need to monitor: 1. Blood pressure 2. Blood glucose levels 3. Urine ketone levels 4. Liver function

1. Blood pressure

A 19-year-old male was started on risperidone. Monitoring for risperidone includes observing for common side effects, including: 1. Bradykinesia, akathisia, and agitation 2. Excessive weight gain 3. Hypertension 4. Potentially fatal agranulocytosis

1. Bradykinesia, akathisia, and agitation

Patients who are on long-term aspirin therapy should have ______ annually. 1. Complete blood count 2. Salicylate level 3. Amylase 4. Urine analysis

1. Complete blood count

The drug of choice for type 2 diabetics is metformin. Metformin: 1. Decreases glycogenolysis by the liver 2. Increases the release of insulin from beta cells 3. Increases intestinal uptake of glucose 4. Prevents weight gain associated with hyperglycemia

1. Decreases glycogenolysis by the liver

GLP-1 agonists: 1. Directly bind to a receptor in the pancreatic beta cell 2. Have been approved for monotherapy 3. Speed gastric emptying to decrease appetite 4. Can be given orally once daily

1. Directly bind to a receptor in the pancreatic beta cell

Which of the following statements about pancreatic enzymes is true? 1. Dosing may be titrated according to the decrease of steatorrhea. 2. The amount of carbohydrates in the meal drives the amount of enzyme used. 3. The amount of medication used is increased with a cystic fibrosis pulmonary flare. 4. The FDA and Internet-available formulations are bioequivalent.

1. Dosing may be titrated according to the decrease of steatorrhea.

Before prescribing metformin, the provider should: 1. Draw a serum creatinine level to assess renal function. 2. Try the patient on insulin. 3. Prescribe a thyroid preparation if the patient needs to lose weight. 4. All of the above

1. Draw a serum creatinine level to assess renal function.

Prior to prescribing metformin, the provider should: 1. Draw a serum creatinine to assess renal function 2. Try the patient on insulin 3. Tell the patient to increase iodine intake 4. Have the patient stop taking any sulfonylurea to avoid dangerous drug interactions

1. Draw a serum creatinine to assess renal function

Taylor is a 10-year-old child diagnosed with major depression. The appropriate first-line antidepressant for children is: 1. Fluoxetine 2. Fluvoxamine 3. Sertraline 4. Escitalopram

1. Fluoxetine

Rabi is being prescribed phenytoin for seizures. Monitoring includes assessing: 1. For phenytoin hypersensitivity syndrome 3 to 8 weeks after starting treatment 2. For pedal edema throughout therapy 3. Heart rate at each visit and consider altering therapy if heart rate is less than 60 bpm 4. For vision changes, such as red-green blindness, at least annually

1. For phenytoin hypersensitivity syndrome 3 to 8 weeks after starting treatment

When the total daily insulin dose is split and given twice daily, which of the following rules may be followed? 1. Give two-thirds of the total dose in the morning and one-third in the evening. 2. Give 0.3 units per kg of premixed 70/30 insulin with one-third in the morning and two-thirds in the evening. 3. Give 50% of an insulin glargine dose in the morning and 50% in the evening. 4. Give long-acting insulin in the morning and short-acting insulin at bedtime.

1. Give two-thirds of the total dose in the morning and one-third in the evening.

The American Heart Association states that people with diabetes have a 2- to 4-fold increase in the risk of dying from cardiovascular disease. Treatments and targets that do not appear to decrease risk for micro- and macro-vascular complications include: 1. Glycemic targets between 7% and 7.5% 2. Use of insulin in type 2 diabetics 3. Control of hypertension and hyperlipidemia 4. Stopping smoking

1. Glycemic targets between 7% and 7.5%

Phil is starting treatment with febuxostat (Uloric). Education of patients starting febuxostat includes: 1. Gout may worsen with therapy. 2. Febuxostat may cause severe diarrhea. 3. He should consume a high-calcium diet. 4. He will need frequent CBC monitoring.

1. Gout may worsen with therapy.

All diabetic patients with hyperlipidemia should be treated with: 1. HMG-CoA reductase inhibitors 2. Fibric acid derivatives 3. Nicotinic acid 4. Colestipol

1. HMG-CoA reductase inhibitors.

Adam has type 1 diabetes and plays tennis for his university. He exhibits a knowledge deficit about his insulin and his diagnosis. He should be taught that: 1. He should increase his carbohydrate intake during times of exercise. 2. Each brand of insulin is equal in bioavailability, so buy the least expensive. 3. Alcohol produces hypoglycemia and can help control his diabetes when taken in small amounts. 4. If he does not want to learn to give himself injections, he may substitute an oral hypoglycemic to control his diabetes.

1. He should increase his carbohydrate intake during times of exercise.

Prior to starting antidepressants, patients should have laboratory testing to rule out: 1. Hypothyroidism 2. Anemia 3. Diabetes mellitus 4. Low estrogen levels

1. Hypothyroidism

Kirk sprained his ankle and is asking for pain medication for his mild-to-moderate pain. The appropriate first-line medication would be: 1. Ibuprofen (Advil) 2. Acetaminophen with hydrocodone (Vicodin) 3. Oxycodone (Oxycontin) 4. Oral morphine (Roxanol)

1. Ibuprofen (Advil)

Bisphosphonates treat or prevent osteoporosis by: 1. Inhibiting osteoclastic activity 2. Fostering bone resorption 3. Enhancing calcium uptake in the bone 4. Strengthening the osteoclastic proton pump

1. Inhibiting osteoclastic activity

Patients who are prescribed olanzapine (Zyprexa) should be monitored for: 1. Insomnia 2. Weight gain 3. Hypertension 4. Galactorrhea

1. Insomnia

Why is the consistency of taking paroxetine (Paxil) and never running out of medication more important than with most other selective serotonin reuptake inhibitors (SSRIs)? 1. It has a shorter half-life and withdrawal syndrome has a faster onset without taper. 2. It has the longest half-life and the withdrawal syndrome has a faster onset. 3. It is quasi-addictive in the dopaminergic reward system. 4. It is the most activating of SSRI medications and will cause the person to have sudden deep sadness.

1. It has a shorter half-life and withdrawal syndrome has a faster onset without taper.

Infants with congenital hypothyroidism are treated with: 1. Levothyroxine 2. Liothyronine 3. Liotrix 4. Methimazole

1. Levothyroxine

Treatment of a patient with hypothyroidism and cardiovascular disease consists of: 1. Levothyroxine 2. Liothyronine 3. Liotrix 4. Methimazole

1. Levothyroxine

Patients with cystic fibrosis are often prescribed enzyme replacement for pancreatic secretions. Each replacement drug has lipase, protease, and amylase components, but the drug is prescribed in units of: 1. Lipase 2. Protease 3. Amylase 4. Pancreatin

1. Lipase

The drugs recommended by the American Academy of Pediatrics for use in children with diabetes (depending upon type of diabetes) are: 1. Metformin and insulin 2. Sulfonylureas and insulin glargine 3. Split-mixed dose insulin and GPL-1 agonists 4. Biguanides and insulin lispro

1. Metformin and insulin

Besides osteoporosis, IV bisphosphonates are also indicated for: 1. Paget's Disease 2. Early osteopenia 3. Renal cancer 4. Early closure of cranial sutures

1. Paget's Disease

Patients need to be questioned about all pain sites because: 1. Patients tend to report the most severe or important in their perception. 2. Pain tolerance generally decreases with repeated exposure. 3. The reported pain site is usually the most important to treat. 4. Pain may be referred from a different site to the one reported.

1. Patients tend to report the most severe or important in their perception.

All nonsteroidal anti-inflammatory drugs (NSAIDS) have an FDA Black Box Warning regarding: 1. Potential for causing life-threatening GI bleeds 2. Increased risk of developing systemic arthritis with prolonged use 3. Risk of life-threatening rashes, including Stevens-Johnson 4. Potential for transient changes in serum glucose

1. Potential for causing life-threatening GI bleeds

Kelly is a 14-year-old patient who presents to the clinic with a classic migraine. She says she is having a headache two to three times a month. The initial plan would be: 1. Prescribe NSAIDs as abortive therapy and have her keep a headache diary to identify her triggers. 2. Prescribe zolmitriptan (Zomig) as abortive therapy and recommend relaxation therapy to reduce her stress. 3. Prescribe acetaminophen with codeine (Tylenol #3) for her to take at the first onset of her migraine. 4. Prescribe sumatriptan (Imitrex) nasal spray and arrange for her to receive the first dose in the clinic.

1. Prescribe NSAIDs as abortive therapy and have her keep a headache diary to identify her triggers.

Kelly is a 14-year-old patient who presents to the clinic with a classic migraine. She says she is having a headache two to three times a month. The initial plan would be: 1. Prescribe NSAIDs as abortive therapy and have her keep a headache diary to identify her triggers. 2. Prescribe zolmitriptan (Zomig) as abortive therapy and recommend relaxation therapy to reduce her stress. 3. Prescribe acetaminophen with codeine (Tylenol #3) for her to take at the first onset of her migraine. 4. Prescribe sumatriptan (Imitrex) nasal spray and arrange for her to receive the first dose in the clinic.

1. Prescribe NSAIDs as abortive therapy and have her keep a headache diary to identify her triggers.

Age is a factor in different responses to pain. Which of the following age-related statements about pain is NOT true? 1. Preterm and newborn infants do not yet have functional pain pathways. 2. Painful experiences and prolonged exposure to analgesic drugs during pregnancy may permanently alter neuronal organization in the child. 3. Increases in the pain threshold in older adults may be related to peripheral neuropathies and changes in skin thickness. 4. Decreases in pain tolerance are evident in older adults.

1. Preterm and newborn infants do not yet have functional pain pathways.

Diabetic autonomic neuropathy (DAN) is the earliest and most common complication of diabetes. Symptoms associated with DAN include: 1. Resting tachycardia, exercise intolerance, and orthostatic hypotension 2. Gastroparesis, cold intolerance, and moist skin 3. Hyperglycemia, erectile dysfunction, and deficiency of free fatty acids 4. Pain, loss of sensation, and muscle weakness

1. Resting tachycardia, exercise intolerance, and orthostatic hypotension

Suzanne is started on paroxetine (Paxil), a selective serotonin reuptake inhibitor (SSRI), for depression. Education regarding her antidepressant includes: 1. SSRIs may take 2 to 6 weeks before she will have maximum drug effects. 2. Red-green color blindness may occur and should be reported. 3. If she experiences dry mouth or heart rates greater than 80, she should stop taking the drug immediately. 4. She should eat lots of food high in fiber to prevent constipation.

1. SSRIs may take 2 to 6 weeks before she will have maximum drug effects.

Patients who are on or who will be starting chronic corticosteroid therapy need monitoring of: 1. Serum glucose 2. Stool culture 3. Folate levels 4. Vitamin B12

1. Serum glucose

Treatment with insulin for type 1 diabetics: 1. Starts with a total daily dose of 0.2 to 0.4 units per kg of body weight 2. Divides the total doses into three injections based on meal size 3. Uses a total daily dose of insulin glargine given once daily with no other insulin required 4. Is based on the level of blood glucose

1. Starts with a total daily dose of 0.2 to 0.4 units per kg of body weight

The decision may be made to switch from twice daily neutral protamine Hagedorn (NPH) insulin to insulin glargine to improve glycemia control throughout the day. If this is done: 1. The initial dose of glargine is reduced by 20% to avoid hypoglycemia. 2. The initial dose of glargine is 2 to 10 units per day. 3. Patients who have been on high doses of NPH will need tests for insulin antibodies. 4. Obese patients may require more than 100 units per day.

1. The initial dose of glargine is reduced by 20% to avoid hypoglycemia.

The longer-term Xanax patient comes in and states they need a higher dose of the medication. They deny any additional, new, or accelerating triggers of their anxiety. What is the probable reason? 1. They have become tolerant of the medication, which is characterized by the need for higher and higher doses. 2. They are a drug seeker. 3. They are suicidal. 4. They only need additional counseling on lifestyle modification.

1. They have become tolerant of the medication, which is characterized by the need for higher and higher doses.

Cara is taking levetiracetam (Keppra) to treat seizures. Routine education for levetiracetam includes reminding her: 1. To not abruptly discontinue levetiracetam due to risk for withdrawal seizures 2. To wear sunscreen due to photosensitivity from levetiracetam 3. To get an annual eye exam while on levetiracetam 4. To report weight loss if it occurs

1. To not abruptly discontinue levetiracetam due to risk for withdrawal seizures

An appropriate first-line drug for the treatment of depression with fatigue and low energy would be: 1. Venlafaxine (Effexor) 2. Escitalopram (Lexapro) 3. Buspirone (Buspar) 4. Amitriptyline (Elavil)

1. Venlafaxine (Effexor)

Patients should be instructed regarding the rapid onset of zolpidem (Ambien) because: 1. Zolpidem should be taken just before going to bed. 2. Zolpidem may cause dry mouth and constipation. 3. Patients may need to double the dose for effectiveness. 4. They should stop drinking alcohol at least 30 minutes before taking zolpidem.

1. Zolpidem should be taken just before going to bed.

Chronic treatment osmotic lacative

10-20g lactulose

Adult dosing for biscodyl

10mg

max dose for loperamide

16mg MAX (4mg PO than 2 mg after each stool)

A woman who is pregnant and has hyperthyroidism is best managed by a specialty team who will most likely treat her with: 1. Methimazole 2. Propylthiouracil (PTU) 3. Radioactive iodine 4. Nothing, treatment is best delayed until after her pregnancy ends

2

After starting a patient with Grave's disease on an antithyroid agent such as methimazole, patient monitoring includes TSH and free T4 every: 1. 1 to 2 weeks 2. 3 to 4 weeks 3. 2 to 3 months 4. 6 to 9 months

2

In addition to methimazole, a symptomatic patient with hyperthyroidism may need a prescription for: 1. A calcium channel blocker 2. A beta blocker 3. Liothyronine 4. An alpha blocker

2

What is the desired mixed of T3 to T4 drug levels in newly diagnosed endocrine patients? 1. 99% of T3 and the rest is T4 to get rapid resolution. 2. Most needs to be T4 to mimic natural ratios of hormone. 3. The ratio is unimportant. 4. The mix needs to be 50-50 at first.

2

When starting a patient on levothyroxine for hypothyroidism the patient will need follow-up measurement of thyroid function in: 1. 2 weeks 2. 4 weeks 3. 2 months 4. 6 months

2

After starting a patient with Grave's disease on an antithyroid agent such as methimazole, patient monitoring includes TSH and free T4 every: 1. 1 to 2 weeks 2. 3 to 4 weeks 3. 2 to 3 months 4. 6 to 9 months

2. 3 to 4 weeks

The trial period to determine effective anti-inflammatory activity when starting a patient on aspirin for rheumatoid arthritis is: 1. 48 hours 2. 4 to 6 days 3. 4 weeks 4. 2 months

2. 4 to 6 days

When starting a patient on levothyroxine for hypothyroidism the patient will need follow-up measurement of thyroid function in: 1. 2 weeks 2. 4 weeks 3. 2 months 4. 6 months

2. 4 weeks

Administration of exenatide is by subcutaneous injection: 1. 30 minutes prior to the morning meal 2. 60 minutes prior to the morning and evening meal 3. 15 minutes after the evening meal 4. 60 minutes before each meal daily

2. 60 minutes prior to the morning and evening meal

When given subcutaneously, how long until neutral protamine Hagedorn insulin begins to take effect (onset of action) after administration? 1. 15 to 30 minutes 2. 60 to 90 minutes 3. 3 to 4 hours 4. 6 to 8 hours

2. 60 to 90 minutes

In addition to methimazole, a symptomatic patient with hyperthyroidism may need a prescription for: 1. A calcium channel blocker 2. A beta blocker 3. Liothyronine 4. An alpha blocker

2. A beta blocker

Type 2 diabetes is a complex disorder involving: 1. Absence of insulin production by the beta cells 2. A suboptimal response of insulin-sensitive tissues in the liver 3. Increased levels of glucagon-like peptide in the postprandial period 4. Too much fat uptake in the intestine

2. A suboptimal response of insulin-sensitive tissues in the liver

Brands of pancreatic enzyme replacement drugs are: 1. Bioequivalent 2. About the same in cost per unit of lipase across brands 3. Able to be interchanged between generic and brand-name products to reduce cost 4. None of the above

2. About the same in cost per unit of lipase across brands

Kasey fractured his ankle in two places and is asking for medication for his pain. The appropriate first-line medication would be: 1. Ibuprofen (Advil) 2. Acetaminophen with hydrocodone (Vicodin) 3. Oxycodone (Oxycontin) 4. Oral morphine (Roxanol)

2. Acetaminophen with hydrocodone (Vicodin)

All diabetic patients with known cardiovascular disease should be treated with: 1. Beta blockers to prevent MIs 2. Angiotensin-converting enzyme inhibitors and aspirin to reduce risk of cardiovascular events 3. Sulfonylureas to decrease cardiovascular mortality 4. Pioglitazone to decrease atherosclerotic plaque buildup

2. Angiotensin-converting enzyme inhibitors and aspirin to reduce risk of cardiovascular events

Sarah, a 42-year-old female, requests a prescription for an anorexiant to treat her obesity. A trial of phentermine is prescribed. Prescribing precautions include: 1. Understanding that obesity is a contraindication to prescribing phentermine 2. Anorexiants may cause tolerance and should only be prescribed for 6 months 3. Patients should be monitored for postural hypotension 4. Renal function should be monitored closely while on anorexiants

2. Anorexiants may cause tolerance and should only be prescribed for 6 months

Sook has been prescribed gabapentin to treat neuropathic pain and is complaining of feeling depressed and having "strange" thoughts. The appropriate initial action would be: 1. Increase her dose 2. Assess for suicidal ideation 3. Discontinue the medication immediately 4. Decrease her dose to half then slowly titrate up the dose

2. Assess for suicidal ideation

Vicky, age 56 years, comes to the clinic requesting a refill of her Fiorinal (aspirin and butalbital) that she takes for migraines. She has been taking this medication for over 2 years for migraines and states one dose usually works to abort her migraine. What is the best care for her? 1. Switch her to sumatriptan (Imitrex) to treat her migraines. 2. Assess how often she is using Fiorinal and refill her medication. 3. Switch her to a beta blocker such as propranolol to prevent her migraine. 4. Request she return to the original prescriber of Fiorinal as you do not prescribe butalbital for migraines.

2. Assess how often she is using Fiorinal and refill her medication.

Vicky, age 56 years, comes to the clinic requesting a refill of her Fiorinal (aspirin and butalbital) that she takes for migraines. She has been taking this medication for over 2 years for migraines and states one dose usually works to abort her migraine. What is the best care for her? 1. Switch her to sumatriptan (Imitrex) to treat her migraines. 2. Assess how often she is using Fiorinal and refill her medication. 3. Switch her to a beta blocker such as propranolol to prevent her migraine. 4. Request she return to the original prescriber of Fiorinal as you do not prescribe butalbital for migraines.

2. Assess how often she is using Fiorinal and refill her medication.

Narcotics are exogenous opiates. They act by: 1. Inhibiting pain transmission in the spinal cord 2. Attaching to receptors in the afferent neuron to inhibit the release of substance P 3. Blocking neurotransmitters in the midbrain 4. Increasing beta-lipoprotein excretion from the pituitary gland

2. Attaching to receptors in the afferent neuron to inhibit the release of substance

Dwayne has recently started on carbamazepine to treat seizures. He comes to see you and you note that while his carbamazepine levels had been in the therapeutic range, they are now low. The possible cause for the low carbamazepine levels include: 1. Dwayne hasn't been taking his carbamazepine because it causes insomnia. 2. Carbamazepine auto-induces metabolism, leading to lower levels in spite of good compliance. 3. Dwayne was not originally prescribed the correct amount of carbamazepine. 4. Carbamazepine is probably not the right antiseizure medication for Dwayne.

2. Carbamazepine auto-induces metabolism, leading to lower levels in spite of good compliance.

Pathological similarities and differences between acute pain and chronic pain include: 1. Both have decreased levels of endorphins. 2. Chronic pain has a predominance of C-neuron stimulation. 3. Acute pain is most commonly associated with irritation of peripheral nerves. 4. Acute pain is diffuse and hard to localize.

2. Chronic pain has a predominance of C-neuron stimulation.

Patient education when prescribing colchicine includes: 1. Colchicine may be constipating. 2. Colchicine always causes some degree of diarrhea. 3. Mild muscle weakness is normal. 4. Moderate amounts of alcohol are safe with colchicine.

2. Colchicine always causes some degree of diarrhea.

Metformin is a primary choice of drug to treat hyperglycemia in type 2 diabetes because it: 1. Substitutes for insulin usually secreted by the pancreas 2. Decreases glycogenolysis by the liver 3. Increases the release of insulin from beta cells 4. Decreases peripheral glucose utilization

2. Decreases glycogenolysis by the liver

An appropriate drug to initially treat panic disorder is: 1. Alprazolam (Xanax) 2. Diazepam (Valium) 3. Buspirone (Buspar) 4. Amitriptyline (Elavil)

2. Diazepam (Valium)

Lispro is an insulin analogue produced by recombinant DNA technology. Which of the following statements about this form of insulin is NOT true? 1. Optimal time of preprandial injection is 15 minutes. 2. Duration of action is increased when the dose is increased. 3. It is compatible with neutral protamine Hagedorn insulin. 4. It has no pronounced peak.

2. Duration of action is increased when the dose is increased.

Tom is taking lithium for bipolar disorder. He should be taught to: 1. Take his lithium with food 2. Eat a diet with consistent levels of salt (sodium) 3. Drink at least 2 quarts of water if he is in a hot environment 4. Monitor blood glucose levels

2. Eat a diet with consistent levels of salt (sodium)

Preventative therapy for cluster headaches includes: 1. Massage or relaxation therapy 2. Ergotamine nightly before bed 3. Intranasal lidocaine four times a day during "clusters" of headaches 4. Propranolol (Inderal) daily

2. Ergotamine nightly before bed

Preventative therapy for cluster headaches includes: 1. Massage or relaxation therapy 2. Ergotamine nightly before bed 3. Intranasal lidocaine four times a day during "clusters" of headaches 4. Propranolol (Inderal) daily

2. Ergotamine nightly before bed

An appropriate drug for the treatment of depression with anxiety would be: 1. Alprazolam (Xanax) 2. Escitalopram (Lexapro) 3. Buspirone (Buspar) 4. Amitriptyline (Elavil)

2. Escitalopram (Lexapro)

Both men and women experience bone loss with aging. The bones most likely to demonstrate significant loss are: 1. Cortical bones 2. Femoral neck bones 3. Cervical vertebrae 4. Pelvic bones

2. Femoral neck bones

Potentially fatal granulocytopenia has been associated with treatment of hyperthyroidism with propylthiouracil. Patients should be taught to report: 1. Tinnitus and decreased salivation 2. Fever and sore throat 3. Hypocalcemia and osteoporosis 4. Laryngeal edema and difficulty swallowing

2. Fever and sore throat

Prevention of conversion from prediabetes to diabetes in young children must take highest priority and should focus on: 1. Aggressive dietary manipulation to prevent obesity 2. Fostering LDL levels less than 100 mg/dl and total cholesterol less than 170 mg/dl to prevent cardiovascular disease 3. Maintaining a blood pressure that is less than 80% based on weight and height to prevent hypertension 4. All of the above

2. Fostering LDL levels less than 100 mg/dl and total cholesterol less than 170 mg/dl to prevent cardiovascular disease

Insulin preparations are divided into categories based on onset, duration, and intensity of action following subcutaneous injection. Which of the following insulin preparations has the shortest onset and duration of action? 1. Lispro 2. Glulisine 3. Glargine 4. Detemir

2. Glulisine

Sulfonylureas may be added to a treatment regimen for type 2 diabetics when lifestyle modifications and metformin are insufficient to achieve target glucose levels. Sulfonylureas have been moved to Step 2 therapy because they: 1. Increase endogenous insulin secretion 2. Have a significant risk for hypoglycemia 3. Address the insulin resistance found in type 2 diabetics 4. Improve insulin binding to receptors

2. Have a significant risk for hypoglycemia

The patient shares with the provider that he is taking his Prozac at night before going to bed. What is the best response? 1. This is a good idea because this class of medications generally makes people sleepy. 2. Have you noticed that you are having more sleep issues since you started that? 3. This a good way to remember to take your daily medications because it is near your toothbrush. 4. This is a good plan because you can eat grapefruit if there is 8-12 hours difference in the time each are ingested.

2. Have you noticed that you are having more sleep issues since you started that?

The American Diabetic Association has recommended which of the following tests for ongoing management of diabetes? 1. Fasting blood glucose 2. HbA1C 3. Thyroid function tests 4. Electrocardiograms

2. HbA1C

David is a 34-year-old patient who is starting on paroxetine (Paxil) for depression. David's education regarding his medication would include: 1. Paroxetine may cause intermittent diarrhea. 2. He may experience sexual dysfunction beginning a month after he starts therapy. 3. He may have constipation and he should increase fluids and fiber. 4. Paroxetine has a long half-life so he may occasionally skip a dose.

2. He may experience sexual dysfunction beginning a month after he starts therapy.

Scott's seizures are well controlled on topiramate and he wants to start playing baseball. Education for Scott regarding his topiramate includes: 1. He should not play sports due to the risk of increased seizures 2. He should monitor his temperature and ability to sweat in the heat while playing 3. Reminding him that he may need higher dosages of topiramate when exercising 4. Encouraging him to use sunscreen due to photosensitivity from topiramate

2. He should monitor his temperature and ability to sweat in the heat while playing

Bart is a patient is a 67-year-old male with T2 DM. He is on glipizide and metformin. He presents to the clinic with confusion, sluggishness, and extreme thirst. His wife tells you Bart does not follow his meal plan or exercise regularly, and hasn't checked his BG for 1 week. A random glucose is drawn and it is 500. What is a likely diagnosis based on preliminary assessment? 1. Diabetic keto acidosis (DKA) 2. Hyperglycemic hyperosmolar syndrome (HHS) 3. Infection 4. Hypoglycemia

2. Hyperglycemic hyperosmolar syndrome (HHS)

One of the main drug classes used to treat acute pain is NSAIDs. They are used because: 1. They have less risk for liver damage than acetaminophen. 2. Inflammation is a common cause of acute pain. 3. They have minimal GI irritation. 4. Regulation of blood flow to the kidney is not affected by these drugs.

2. Inflammation is a common cause of acute pain.

Unlike most type 2 diabetics where obesity is a major issue, older adults with low body weight have higher risks for morbidity and mortality. The most reliable indicator of poor nutritional status in older adults is: 1. Weight loss in previously overweight persons 2. Involuntary loss of 10% of body weight in less than 6 months 3. Decline in lean body mass over a 12-month period 4. Increase in central versus peripheral body adiposity

2. Involuntary loss of 10% of body weight in less than 6 months

What would one expected assessment finding be for hyperglycemic hyperosmolar syndrome? 1. Low hemoglobin 2. Ketones in the urine 3. Deep, labored breathing 4. pH of 7.35

2. Ketones in the urine RATIONALE: Normal pH of 7.35. C and B are indicative of DKA. Hgb may be low, normal or high in HHNKS.

Henry presents to clinic with a significantly swollen, painful great toe and is diagnosed with gout. Of the following, which would be the best treatment for Henry? 1. High-dose colchicine 2. Low-dose colchicine 3. High-dose aspirin 4. Acetaminophen with codeine

2. Low-dose colchicine

What is the desired mixed of T3 to T4 drug levels in newly diagnosed endocrine patients? 1. 99% of T3 and the rest is T4 to get rapid resolution. 2. Most needs to be T4 to mimic natural ratios of hormone. 3. The ratio is unimportant. 4. The mix needs to be 50-50 at first.

2. Most needs to be T4 to mimic natural ratios of hormone.

Chemical dependency assessment is integral to the initial assessment of chronic pain. Which of the following raises a "red flag" about potential chemical dependency? 1. Use of more than one drug to treat the pain 2. Multiple times when prescriptions are lost with requests to refill 3. Preferences for treatments that include alternative medicines 4. Presence of a family member who has abused drugs

2. Multiple times when prescriptions are lost with requests to refill

A first-line drug for abortive therapy in simple migraine is: 1. Sumatriptan (Imitrex) 2. Naproxen (Aleve) 3. Butorphanol nasal spray (Stadol NS) 4. Butalbital and acetaminophen (Fioricet)

2. Naproxen (Aleve)

A first-line drug for abortive therapy in simple migraine is: 1. Sumatriptan (Imitrex) 2. Naproxen (Aleve) 3. Butorphanol nasal spray (Stadol NS) 4. Butalbital and acetaminophen (Fioricet)

2. Naproxen (Aleve)

Juanita presents to the clinic with a complaint of headaches off and on for months. She reports they feel like someone is "squeezing" her head. She occasionally takes Tylenol for the pain, but usually just "toughs it out." Initial treatment for tension headache includes asking her to keep a headache diary and a prescription for: 1. Sumatriptan (Imitrex) 2. Naproxen (Aleve) 3. Ergotamine (Ergostat) 4. Tylenol with codeine (Tylenol #3)

2. Naproxen (Aleve)

Juanita presents to the clinic with a complaint of headaches off and on for months. She reports they feel like someone is "squeezing" her head. She occasionally takes Tylenol for the pain, but usually just "toughs it out." Initial treatment for tension headache includes asking her to keep a headache diary and a prescription for: 1. Sumatriptan (Imitrex) 2. Naproxen (Aleve) 3. Ergotamine (Ergostat) 4. Tylenol with codeine (Tylenol #3)

2. Naproxen (Aleve)

Routine screening of asymptomatic adults for diabetes is appropriate for: 1. Individuals who are older than 45 and have a BMI of less than 25 kg/m2 2. Native Americans, African Americans, and Hispanics 3. Persons with HDL cholesterol greater than 100 mg/dl 4. Persons with prediabetes confirmed on at least two occasions

2. Native Americans, African Americans, and Hispanics

Patients whose total dose of prednisone will exceed 1 gram will most likely need a second prescription for: 1. Metformin, a biguanide to prevent diabetes 2. Omeprazole, a proton pump inhibitor to prevent peptic ulcer disease 3. Naproxen, an NSAID to treat joint pain 4. Furosemide, a diuretic to treat fluid retention

2. Omeprazole, a proton pump inhibitor to prevent peptic ulcer disease

Opiates are used mainly to treat moderate to severe pain. Which of the following is NOT true about these drugs? 1. All opiates are scheduled drugs which require a DEA license to prescribe. 2. Opiates stimulate only mu receptors for the control of pain. 3. Most of the adverse effects of opiates are related to mu receptor stimulation. 4. Naloxone is an antagonist to opiates.

2. Opiates stimulate only mu receptors for the control of pain.

The Pain Management Contract is appropriate for: 1. Patients with cancer who are taking morphine 2. Patients with chronic pain who will require long-term use of opiates 3. Patients who have a complex drug regimen 4. Patients who see multiple providers for pain control

2. Patients with chronic pain who will require long-term use of opiates

Anticholinergic agents, such as benztropine (Cogentin), may be given with a phenothiazine to: 1. Reduce the chance of tardive dyskinesia 2. Potentiate the effects of the drug 3. Reduce the tolerance that tends to occur 4. Increase central nervous system (CNS) depression

2. Potentiate the effects of the drug

Xi, a 54-year-old female, has a history of migraines that do not respond well to OTC migraine medication. She is asking to try Maxalt (rizatriptan) because it works well for her friend. Appropriate decision making would be: 1. Prescribe the Maxalt, but only give her four tablets with no refills to monitor the use. 2. Prescribe Maxalt and arrange to have her observed in the clinic or urgent care with the first dose. 3. Explain that rizatriptan is not used for postmenopausal migraines and recommend Fiorinal (aspirin and butalbital). 4. Prescribe sumatriptan (Imitrex) with the explanation that it is the most effective triptan.

2. Prescribe Maxalt and arrange to have her observed in the clinic or urgent care with the first dose

Xi, a 54-year-old female, has a history of migraines that do not respond well to OTC migraine medication. She is asking to try Maxalt (rizatriptan) because it works well for her friend. Appropriate decision making would be: 1. Prescribe the Maxalt, but only give her four tablets with no refills to monitor the use. 2. Prescribe Maxalt and arrange to have her observed in the clinic or urgent care with the first dose. 3. Explain that rizatriptan is not used for postmenopausal migraines and recommend Fiorinal (aspirin and butalbital). 4. Prescribe sumatriptan (Imitrex) with the explanation that it is the most effective triptan.

2. Prescribe Maxalt and arrange to have her observed in the clinic or urgent care with the first dose.

A woman who is pregnant and has hyperthyroidism is best managed by a specialty team who will most likely treat her with: 1. Methimazole 2. Propylthiouracil (PTU) 3. Radioactive iodine 4. Nothing, treatment is best delayed until after her pregnancy ends

2. Propylthiouracil (PTU)

Antonia is a 3-year-old child who has a history of status epilepticus. Along with her routine antiseizure medication, she should also have a home prescription for_________ to be used for an episode of status epilepticus. 1. IV phenobarbital 2. Rectal diazepam (Diastat) 3. IV phenytoin (Dilantin) 4. Oral carbamazepine (Tegretol)

2. Rectal diazepam (Diastat)

Jaycee has been on escitalopram (Lexapro) for a year and is willing to try tapering off of the selective serotonin reuptake inhibitor. What is the initial dosage adjustment when starting a taper off antidepressants? 1. Change dose to every other day dosing for a week 2. Reduce dose by 50% for 3 to 4 days 3. Reduce dose by 50% every other day 4. Escitalopram (Lexapro) can be stopped abruptly due to its long half-life

2. Reduce dose by 50% for 3 to 4 days

Patients who are on chronic long-term corticosteroid therapy need education regarding: 1. Receiving all vaccinations, especially the live flu vaccine 2. Reporting black tarry stools or abdominal pain 3. Eating a high carbohydrate diet with plenty of fluids 4. Small amounts of alcohol are generally tolerated.

2. Reporting black tarry stools or abdominal pain

Zainab is taking lamotrigine (Lamictal) and presents to the clinic with fever and lymphadenopathy. Initial evaluation and treatment includes: 1. Reassuring her she has a viral infection and to call if she isn't better in 4 or 5 days 2. Ruling out a hypersensitivity reaction that may lead to multi-organ failure 3. Rapid strep test and symptomatic care if strep test is negative 4. Observation only, with further assessment if she worsens

2. Ruling out a hypersensitivity reaction that may lead to multi-organ failure

Elderly patients who are started on levothyroxine for thyroid replacement should be monitored for: 1. Excessive sedation 2. Tachycardia and angina 3. Weight gain 4. Cold intolerance

2. Tachycardia and angina

When prescribing temazepam (Restoril) for insomnia, patient education includes: 1. Take temazepam nightly approximately 15 minutes before bedtime. 2. Temazepam should not be used more than three times a week for less than 3 months. 3. Drinking 1 ounce of alcohol will cause additive effects and the patient will sleep better. 4. Exercise for at least 30 minutes within 2 hours of bedtime to enhance the effects of temazepam.

2. Temazepam should not be used more than three times a week for less than 3 months.

What "onset of action" symptoms should be reviewed with patients who have been newly prescribed a selective serotonin reuptake inhibitor? 1. They will have insomnia for a week. 2. They can feel a bit of nausea, but this resolves in a week. 3. They will have an "onset seizure" but this is considered normal. 4. They will no longer dream.

2. They can feel a bit of nausea, but this resolves in a week.

What is the role of calcium supplements when patients take bisphosphonates? 1. They must be restricted to allow the medication to work. 2. They must be taken in sufficient amounts to provide foundational elements for bone growth. 3. They must be taken at the same time as the bisphosphonates. 4. They only work with bisphosphonates if daily intake is restricted.

2. They must be taken in sufficient amounts to provide foundational elements for bone growth.

Jake, a 45-year-old patient with schizophrenia, was recently hospitalized for acute psychosis due to medication noncompliance. He was treated with IM long-acting haloperidol. Besides monitoring his schizophrenia symptoms, the patient should be assessed by his primary care provider: 1. For excessive weight loss 2. With the Abnormal Involuntary Movement Scale (AIMS) for extrapyramidal symptoms (EPS) 3. Monthly for tolerance to the haloperidol 4. Only by the mental health provider, as most NPs in primary care do not care for mentally ill patients

2. With the Abnormal Involuntary Movement Scale (AIMS) for extrapyramidal symptoms (EPS)

onset of lactulose

24-48hours

In hyperthyroid states, what organ system other than CV must be evaluated to establish potential adverse issues? 1. The liver 2. The nails and skin 3. The eye 4. The ear

3

Laboratory values are actually different for TSH when screening for thyroid issues and when used for medication management. Which of the follow holds true? 1. Screening TSH has a wider range of normal values 0.02-5.0; therapeutic levels need to remain above 5.0. 2. Screening values are much narrower than the acceptable range used to keep a person stable on hormone replacement. 3. Therapeutic values are kept between 0.05 and 3.0 ideally. Screening values are considered acceptable up to 10. 4. Screening values are between 5 and 10, and therapeutic values are greater than 10.

3

What happens to the typical hormone replacement dose when a woman becomes pregnant? 1. Most women need less medication. 2. Most women do not require a dose change. 3. The average woman needs more medication during pregnancy. 4. The average woman needs more medication only if carrying multiples.

3

When starting a patient with hypothyroidism on thyroid replacement hormones patient education would include: 1. They should feel symptomatic improvement in 1 to 2 weeks. 2. Drug adverse effects such as lethargy and dry skin may occur. 3. It may take 4 to 8 weeks to get to euthyroid symptomatically and by laboratory testing. 4. Because of its short half-life, levothyroxine doses should not be missed.

3

A patient on metformin and glipizide arrives at her 11:30 a.m. clinic appointment diaphoretic and dizzy. She reports taking her medication this morning and ate a bagel and coffee for breakfast. BP is 110/70 and random finger-stick glucose is 64. How should this patient be treated? 1. 12 oz apple juice with 1 tsp sugar 2. 10 oz diet soda 3. 8 oz milk or 4 oz orange juice 4. 4 cookies and 8 oz chocolate milk

3. 8 oz milk or 4 oz orange juice RATIONALE: Approximately 15 grams of fast-acting CHO is the treatment of choice. Diet soda would not treat hypoglycemia. The other choices would overtreat and cause rebound hyperglycemia.

Which of the following should not be taken with a selective serotonin reuptake inhibitor? 1. Aged blue cheese 2. Grapefruit 3. Alcohol 4. Green leafy vegetables

3. Alcohol

In choosing a benzodiazepam to treat anxiety the prescriber needs to be aware of the possibility of dependence. The benzodiazepam with the greatest likelihood of rapidly developing dependence is: 1. Chlordiazepoxide (Librium) 2. Clonazepam (Klonopin) 3. Alprazolam (Xanax) 4. Oxazepam (Serax)

3. Alprazolam (Xanax)

Six-year-old Lucy has recently been started on ethosuximide (Zarontin) for seizures. She should be monitored for: 1. Increased seizure activity, as this drug may auto-induce seizures 2. Altered renal function, including renal failure 3. Blood dyscrasias, which are uncommon but possible 4. Central nervous system excitement, leading to insomnia

3. Blood dyscrasias, which are uncommon but possible

Control targets for patients with diabetes include: 1. HbA1C between 7 and 8 2. Fasting blood glucose levels between 100 and 120 mg/dl 3. Blood pressure less than 130/80 mm Hg 4. LDL lipids less than 130 mg/dl

3. Blood pressure less than 130/80 mm Hg

Both angiotensin converting enzyme inhibitors and some angiotensin II receptor blockers have been approved in treating: 1. Hypertension in diabetic patients 2. Diabetic nephropathy 3. Both 1 and 2 4. Neither 1 nor 2

3. Both 1 and 2

Ethnic groups differ in their risk for and presentation of diabetes. Hispanics: 1. Have a high incidence of obesity, elevated triglycerides, and hypertension 2. Do best with drugs that foster weight loss, such as metformin 3. Both 1 and 2 4. Neither 1 nor 2

3. Both 1 and 2

Sitagliptin has been approved for: 1. Monotherapy in once-daily doses 2. Combination therapy with metformin 3. Both 1 and 2 4. Neither 1 nor 2

3. Both 1 and 2

An appropriate first-line drug to try for mild to moderate generalized anxiety disorder would be: 1. Alprazolam (Xanax) 2. Diazepam (Valium) 3. Buspirone (Buspar) 4. Amitriptyline (Elavil)

3. Buspirone (Buspar)

Which of the following is not an indication that growth hormone supplements should be discontinued? 1. Imaging indication of epiphyseal closure 2. Growth curve increases have plateaued 3. Complaints of mild bone pain 4. Achievement of anticipated height goals

3. Complaints of mild bone pain

Carbamazepine has a Black Box Warning due to life-threatening: 1. Renal toxicity, leading to renal failure 2. Hepatotoxicity, leading to liver failure 3. Dermatologic reaction, including Steven's Johnson and toxic epidermal necrolysis 4. Cardiac effects, including supraventricular tachycardia

3. Dermatologic reaction, including Steven's Johnson and toxic epidermal necrolysis

Daniel has been on 60 mg of prednisone for 10 days to treat a severe asthma exacerbation. It is time to discontinue the prednisone. How is prednisone discontinued? 1. Patients with asthma are transitioned directly off the prednisone onto inhaled corticosteroids. 2. Prednisone can be abruptly discontinued with no adverse effects. 3. Develop a tapering schedule to slowly wean Daniel off the prednisone. 4. Substitute the prednisone with another anti-inflammatory such as ibuprofen.

3. Develop a tapering schedule to slowly wean Daniel off the prednisone.

Hypoglycemia can result from the action of either insulin or an oral hypoglycemic. Signs and symptoms of hypoglycemia include: 1. "Fruity" breath odor and rapid respiration 2. Diarrhea, abdominal pain, weight loss, and hypertension 3. Dizziness, confusion, diaphoresis, and tachycardia 4. Easy bruising, palpitations, cardiac dysrhythmias, and coma

3. Dizziness, confusion, diaphoresis, and tachycardia

One major drug used to treat bipolar disease is lithium. Because lithium has a narrow therapeutic range, it is important to recognize symptoms of toxicity, such as: 1. Orthostatic hypotension 2. Agitation and irritability 3. Drowsiness and nausea 4. Painful urination and abdominal distention

3. Drowsiness and nausea

Allison is an 18-year-old college student with type 1 diabetes. Allison's pre-meal BG at 11:30 a.m. is 130. She eats an apple and has a sugar-free soft drink. At 1 p.m. before swimming her BG is 80. What should she do? 1. Proceed with the swimming class. 2. Recheck her BG immediately. 3. Eat a granola bar or other snack with CHO. 4. Take an additional dose of insulin.

3. Eat a granola bar or other snack with CHO.

Prophylactic use of bisphosphonates is recommended for patients with early osteopenia related to long-term use of which of the following drugs? 1. Selective estrogen receptor modulators 2. Aspirin 3. Glucocorticoids 4. Calcium supplements

3. Glucocorticoids

Jamie has fractured his ankle and has received a prescription for acetaminophen and hydrocodone (Vicodin). Education when prescribing Vicodin includes: 1. It is okay to double the dose of Vicodin if the pain is severe. 2. Vicodin is not habit-forming. 3. He should not take any other acetaminophen-containing medications. 4. Vicodin may cause diarrhea; increase his fluid intake.

3. He should not take any other acetaminophen-containing medications.

Insulin is used to treat both types of diabetes. It acts by: 1. Increasing beta cell response to low blood-glucose levels 2. Stimulating hepatic glucose production 3. Increasing peripheral glucose uptake by skeletal muscle and fat 4. Improving the circulation of free fatty acids

3. Increasing peripheral glucose uptake by skeletal muscle and fat

When starting a patient with hypothyroidism on thyroid replacement hormones patient education would include: 1. They should feel symptomatic improvement in 1 to 2 weeks. 2. Drug adverse effects such as lethargy and dry skin may occur. 3. It may take 4 to 8 weeks to get to euthyroid symptomatically and by laboratory testing. 4. Because of its short half-life, levothyroxine doses should not be missed.

3. It may take 4 to 8 weeks to get to euthyroid symptomatically and by laboratory testing.

Josefina is a 2-year-old child with acute otitis media and an upper respiratory infection. Along with an antibiotic she receives a recommendation to treat the ear pain with ibuprofen. What education would her parent need regarding ibuprofen? 1. They can cut an adult ibuprofen tablet in half to give Josefina. 2. The ibuprofen dose can be doubled for severe pain. 3. Josefina needs to be well-hydrated while taking ibuprofen. 4. Ibuprofen is completely safe in children with no known adverse effects.

3. Josefina needs to be well-hydrated while taking ibuprofen.

Dipeptidyl peptidase-4 inhibitors (gliptins) act on the incretin system to improve glycemic control. Advantages of these drugs include: 1. Better reduction in glucose levels than other classes 2. Less weight gain than sulfonylureas 3. Low risk for hypoglycemia 4. Can be given twice daily

3. Low risk for hypoglycemia

Different areas of the brain are involved in specific aspects of pain. The reticular and limbic systems in the brain influence the: 1. Sensory aspects of pain 2. Discriminative aspects of pain 3. Motivational aspects of pain 4. Cognitive aspects of pain

3. Motivational aspects of pain

When blood glucose levels are difficult to control in type 2 diabetes some form of insulin may be added to the treatment regimen to control blood glucose and limit complication risks. Which of the following statements is accurate based on research? 1. Premixed insulin analogues are better at lowering HbA1C and have less risk for hypoglycemia. 2. Premixed insulin analogues and the newer premixed insulins are associated with more weight gain than the oral antidiabetic agents. 3. Newer premixed insulins are better at lowering HbA1C and postprandial glucose levels than long-acting insulins. 4. Patients who are not controlled on oral agents and have postprandial hyperglycemia can have neutral protamine Hagedorn insulin added at bedtime.

3. Newer premixed insulins are better at lowering HbA1C and postprandial glucose levels than long-acting insulins.

Sallie has been taking 10 mg per day of prednisone for the past 6 months. She should be assessed for: 1. Gout 2. Iron deficiency anemia 3. Osteoporosis 4. Renal dysfunction

3. Osteoporosis

James has been diagnosed with cluster headaches. Appropriate acute therapy would be: 1. Butalbital and aspirin (Fiorinal) 2. Meperidine IM (Demerol) 3. Oxygen 100% for 15 to 30 minutes 4. Indomethacin (Indocin)

3. Oxygen 100% for 15 to 30 minu

James has been diagnosed with cluster headaches. Appropriate acute therapy would be: 1. Butalbital and aspirin (Fiorinal) 2. Meperidine IM (Demerol) 3. Oxygen 100% for 15 to 30 minutes 4. Indomethacin (Indocin)

3. Oxygen 100% for 15 to 30 minutes

Jayla is a 9-year-old patient who has been diagnosed with migraines for almost 2 years. She is missing up to a week of school each month. Her headache diary confirms she averages four or five migraines per month. Which of the following would be appropriate? 1. Prescribe amitriptyline (Elavil) daily, start at a low dose and increase dosage slowly every 2 weeks until it's effective in eliminating migraines. 2. Encourage her mother to give her Excedrin Migraine (aspirin, acetaminophen, and caffeine) at the first sign of a headache to abort the headache. 3. Prescribe propranolol (Inderal) to be taken daily for at least 3 months. 4. Explain that it is rare for a 9-year-old child to get migraines and she needs an MRI to rule out a brain tumor.

3. Prescribe propranolol (Inderal) to be taken daily for at least 3 months.

Jayla is a 9-year-old patient who has been diagnosed with migraines for almost 2 years. She is missing up to a week of school each month. Her headache diary confirms she averages four or five migraines per month. Which of the following would be appropriate? 1. Prescribe amitriptyline (Elavil) daily, start at a low dose and increase dosage slowly every 2 weeks until it's effective in eliminating migraines. 2. Encourage her mother to give her Excedrin Migraine (aspirin, acetaminophen, and caffeine) at the first sign of a headache to abort the headache. 3. Prescribe propranolol (Inderal) to be taken daily for at least 3 months. 4. Explain that it is rare for a 9-year-old child to get migraines and she needs an MRI to rule out a brain tumor.

3. Prescribe propranolol (Inderal) to be taken daily for at least 3 months.

When prescribing any headache therapy, appropriate use of medications needs to be discussed to prevent medication-overuse headaches. A clinical characteristic of medication- overuse headaches is that they: 1. Are increasing in frequency 2. Are increasing in intensity 3. Recur when medication wears off 4. Begin to "cluster" into a pattern

3. Recur when medication wears off

When prescribing any headache therapy, appropriate use of medications needs to be discussed to prevent medication-overuse headaches. A clinical characteristic of medication-overuse headaches is that they: 1. Are increasing in frequency 2. Are increasing in intensity 3. Recur when medication wears off 4. Begin to "cluster" into a pattern

3. Recur when medication wears off

Samantha is taking lamotrigine (Lamictal) for her seizures and requests a prescription for combined oral contraceptives (COCs), which interact with lamotrigine and may cause: 1. Contraceptive failure 2. Excessive weight gain 3. Reduced lamotrigine levels, requiring doubling the dose of lamotrigine 4. Induction of estrogen metabolism, requiring higher estrogen content OCs be prescribed

3. Reduced lamotrigine levels, requiring doubling the dose of lamotrigine

The goal of treatment of acute pain is: 1. Pain at a tolerable level where the patient may return to activities of daily living 2. Reduction of pain with a minimum of drug adverse effects 3. Reduction or elimination of pain with minimum adverse reactions 4. Adequate pain relief without constipation or nausea from the drugs

3. Reduction or elimination of pain with minimum adverse reactions

Which of the following statements is true about acute pain? 1. Somatic pain comes from body surfaces and is only sharp and well-localized. 2. Visceral pain comes from the internal organs and is most responsive to acetaminophen and opiates. 3. Referred pain is present in a distant site for the pain source and is based on activation of the same spinal segment as the actual pain site. 4. Acute neuropathic pain is caused by lack of blood supply to the nerves in a given area.

3. Referred pain is present in a distant site for the pain source and is based on activation of the same spinal segment as the actual pain site.

Chronic pain is a complex problem. Some specific strategies to deal with it include: 1. Telling the patient to "let pain be your guide" to using treatment therapies 2. Prescribing pain medication on a "PRN" basis to keep down the amount used 3. Scheduling return visits on a regular basis rather than waiting for poor pain control to drive the need for an appointment 4. All of the above

3. Scheduling return visits on a regular basis rather than waiting for poor pain control to drive the need for an appointment

When prescribing for migraines, patient education includes: 1. Triptans are safe to be used as often as needed as long as the patient is healthy. 2. Use triptan before trying OTC meds such as acetaminophen or naproxen. 3. Stress reduction and regular sleep are integral to migraine treatment. 4. If migraines worsen they are to increase their medication.

3. Stress reduction and regular sleep are integral to migraine treatment.

When prescribing for migraines, patient education includes: 1. Triptans are safe to be used as often as needed as long as the patient is healthy. 2. Use triptan before trying OTC meds such as acetaminophen or naproxen. 3. Stress reduction and regular sleep are integral to migraine treatment. 4. If migraines worsen they are to increase their medication.

3. Stress reduction and regular sleep are integral to migraine treatment.

What happens to the typical hormone replacement dose when a woman becomes pregnant? 1. Most women need less medication. 2. Most women do not require a dose change. 3. The average woman needs more medication during pregnancy. 4. The average woman needs more medication only if carrying multiples.

3. The average woman needs more medication during pregnancy.

In hyperthyroid states, what organ system other than CV must be evaluated to establish potential adverse issues? 1. The liver 2. The nails and skin 3. The eye 4. The ear

3. The eye

Before prescribing phentermine to Sarah, a thorough drug history should be taken including assessing for the use of serotonergic agents such as selective serotonin reuptake inhibitors (SSRIs) and St John's wort due to: 1. Additive respiratory depression risk 2. Additive effects affecting liver function 3. The risk of serotonin syndrome 4. The risk of altered cognitive functioning

3. The risk of serotonin syndrome

Laboratory values are actually different for TSH when screening for thyroid issues and when used for medication management. Which of the follow holds true? 1. Screening TSH has a wider range of normal values 0.02-5.0; therapeutic levels need to remain above 5.0. 2. Screening values are much narrower than the acceptable range used to keep a person stable on hormone replacement. 3. Therapeutic values are kept between 0.05 and 3.0 ideally. Screening values are considered acceptable up to 10. 4. Screening values are between 5 and 10, and therapeutic values are greater than 10.

3. Therapeutic values are kept between 0.05 and 3.0 ideally. Screening values are considered acceptable up to 10.

When prescribing ergotamine suppositories (Wigraine) to treat acute migraine, patient education would include: 1. Ergotamine will briefly make the migraine worse before the migraine resolves. 2. The patient may experience bradycardia and dizziness. 3. They may need premedication with an antinausea medication. 4. Ergotamine works best if the patient starts off with a full suppository to get the full effect.

3. They may need premedication with an antinausea medication.

When prescribing ergotamine suppositories (Wigraine) to treat acute migraine, patient education would include: 1. Ergotamine will briefly make the migraine worse before the migraine resolves. 2. The patient may experience bradycardia and dizziness. 3. They may need premedication with an antinausea medication. 4. Ergotamine works best if the patient starts off with a full suppository to get the full effect.

3. They may need premedication with an antinausea medication.

Studies have shown that control targets that reduce the HbA1C to less than 7% are associated with fewer long-term complications of diabetes. Patients who should have such a target include: 1. Those with long-standing diabetes 2. Older adults 3. Those with no significant cardiovascular disease 4. Young children who are early in their disease

3. Those with no significant cardiovascular disease

Adult dosing for Phenolphthalein

30-60mg

Goals when treating hypothyroidism with thyroid replacement include: 1. Normal TSH and free T4 levels 2. Resolution of fatigue 3. Weight loss to baseline 4. All of the above

4

Once a patient who is being treated for hypothyroidism returns to euthyroid with normal TSH levels, he or she should be monitored with TSH and free T4 levels every: 1. 2 weeks 2. 4 weeks 3. 2 months 4. 6 months

4

When methimazole is started for hyperthyroidism it may take ________ to see a total reversal of hyperthyroid symptoms. 1. 2 to 4 weeks 2. 1 to 2 months 3. 3 to 4 months 4. 6 to 12 months

4

Why are "natural" thyroid products not readily prescribed for most patients? 1. There is no reliability for the amount of hormone per dose. 2. There is higher incidence of allergic reactions. 3. There is a more reliable dose of T3 to T4 per batch. 4. All of the above

4

lomotil is given

4 times a day

The trial period to determine effective anti-inflammatory activity when starting a patient on aspirin for rheumatoid arthritis is:

4 to 6 days

Dose range for sulfasalazine

4-6gm daily

Once a patient who is being treated for hypothyroidism returns to euthyroid with normal TSH levels, he or she should be monitored with TSH and free T4 levels every: 1. 2 weeks 2. 4 weeks 3. 2 months 4. 6 months

4. 6 months

When methimazole is started for hyperthyroidism it may take ________ to see a total reversal of hyperthyroid symptoms. 1. 2 to 4 weeks 2. 1 to 2 months 3. 3 to 4 months 4. 6 to 12 months

4. 6 to 12 months

Which of the following statements is true about age and pain? 1. Use of drugs that depend heavily on the renal system for excretion may require dosage adjustments in very young children. 2. Among the NSAIDs, indomethacin is the preferred drug because of lower adverse effects profiles than other NSAIDs. 3. Older adults who have dementia probably do not experience much pain due to loss of pain receptors in the brain. 4. Acetaminophen is especially useful in both children and adults because it has no effect on platelets and has fewer adverse effects than NSAIDs.

4. Acetaminophen is especially useful in both children and adults because it has no effect on platelets and has fewer adverse effects than NSAIDs.

The action of "gliptins" is different from other antidiabetic agents because they: 1. Have a low risk for hypoglycemia 2. Are not associated with weight gain 3. Close ATP-dependent potassium channels in the beta cell 4. Act on the incretin system to indirectly increase insulin production

4. Act on the incretin system to indirectly increase insulin production

A 66-year-old male was prescribed phenelzine (Nardil) while in an acute psychiatric unit for recalcitrant depression. The NP managing his primary health care needs to understand the following regarding phenelzine and other monoamine oxidase inhibitors (MAOIs): 1. He should not be prescribed any serotonergic drug such as sumatriptan (Imitrex) 2. MAOIs interact with many common foods, including yogurt, sour cream, and soy sauce 3. Symptoms of hypertensive crisis (headache, tachycardia, sweating) require immediate treatment 4. All of the above

4. All of the above

A treatment plan for management of chronic pain should include: 1. Negotiation with the patient to set personal goals for pain management 2. Discussion of ways to improve sleep and stress 3. An exercise program to improve function and fitness 4. All of the above

4. All of the above

Amber is a 24-year-old patient who has had migraines for 10 years. She reports a migraine on average of once a month. The migraines are effectively aborted with naratriptan (Amerge). When refilling Amber's naratriptan, education would include: 1. Naratriptan will interact with antidepressants, including selective serotonin reuptake inhibitors (SSRIs) and St John's wort, and she should inform any providers she sees that she has migraines. 2. Continue to monitor her headaches, if the migraine is consistently happening around her menses there is preventive therapy available. 3. Pregnancy is contraindicated when taking a triptan. 4. All of the above

4. All of the above

Amber is a 24-year-old patient who has had migraines for 10 years. She reports a migraine on average of once a month. The migraines are effectively aborted with naratriptan (Amerge). When refilling Amber's naratriptan, education would include: 1. Naratriptan will interact with antidepressants, including selective serotonin reuptake inhibitors (SSRIs) and St John's wort, and she should inform any providers she sees that she has migraines. 2. Continue to monitor her headaches, if the migraine is consistently happening around her menses there is preventive therapy available. 3. Pregnancy is contraindicated when taking a triptan. 4. All of the above

4. All of the above

Avoid concurrent administration of exenatide with which of the following drugs? 1. Digoxin 2. Warfarin 3. Lovastatin 4. All of the above

4. All of the above

Goals when treating hypothyroidism with thyroid replacement include: 1. Normal TSH and free T4 levels 2. Resolution of fatigue 3. Weight loss to baseline 4. All of the above

4. All of the above

Monitoring a patient on a high-dose aspirin level includes: 1. Salicylate level 2. Complete blood count 3. Urine pH 4. All of the above

4. All of the above

Monitoring for a child on methylphenidate for attention deficit hyperactivity disorder (ADHD) includes: 1. ADHD symptoms 2. Routine height and weight checks 3. Amount of methylphenidate being used 4. All of the above

4. All of the above

Nonpharmacologic therapy for tension headaches includes: 1. Biofeedback 2. Stress management 3. Massage therapy 4. All of the above

4. All of the above

Nonpharmacologic therapy for tension headaches includes: 1. Biofeedback 2. Stress management 3. Massage therapy 4. All of the above

4. All of the above

Pain assessment to determine adequacy of pain management is important for all patients. This assessment is done to: 1. Determine if the diagnosis of source of pain is correct 2. Determine if the current regimen is adequate or different combinations of drugs and non-drug therapy are required 3. Determine if the patient is willing and able to be an active participant in his or her pain management 4. All of the above

4. All of the above

Patients with rheumatoid arthritis who are on chronic low-dose prednisone will need co-treatment with which medications to prevent further adverse effects? 1. A bisphosphonate 2. Calcium supplementation 3. Vitamin D 4. All of the above

4. All of the above

When prescribing an opioid analgesic such as acetaminophen and codeine (Tylenol #3), instructions to the patient should include: 1. The medication may cause sedation and they should not drive. 2. Constipation is a common side effect and they should increase fluids and fiber. 3. Patients should not take any other acetaminophen-containing medications at the same time. 4. All of the above

4. All of the above

Why are "natural" thyroid products not readily prescribed for most patients? 1. There is no reliability for the amount of hormone per dose. 2. There is higher incidence of allergic reactions. 3. There is a more reliable dose of T3 to T4 per batch. 4. All of the above

4. All of the above

Screening for children who meet the following criteria should begin at age 10 and occur every 3 years thereafter: 1. BMI above the 85th percentile for age and sex 2. Family history of diabetes in first- or second-degree relative 3. Hypertension based on criteria for children 4. Any of the above

4. Any of the above

Larry is taking allopurinol to prevent gout. Monitoring of a patient who is taking allopurinol includes: 1. Complete blood count 2. Blood glucose 3. C-reactive protein 4. BUN, creatinine, and creatinine clearance

4. BUN, creatinine, and creatinine clearance

A patient with anxiety and depression may respond to: 1. Duloxetine (Cymbalta) 2. Fluoxetine (Prozac) 3. Oxazepam (Serax) 4. Buspirone (Buspar) and an SSRI combined

4. Buspirone (Buspar) and an SSRI combined

Allison is an 18-year-old college student with type 1 diabetes. She is on NPH twice daily and Novolog before meals. She usually walks for 40 minutes each evening as part of her exercise regimen. She is beginning a 30-minute swimming class three times a week at 1 p.m. What is important for her to do with this change in routine? 1. Delay eating the midday meal until after the swimming class. 2. Increase the morning dose of NPH insulin on days of the swimming class. 3. Adjust the morning insulin injection so that the peak occurs while swimming. 4. Check glucose level before, during, and after swimming.

4. Check glucose level before, during, and after swimming.

Long-term monitoring of patients who are taking carbamazepine includes: 1. Routine troponin levels to assess for cardiac damage 2. Annual eye examinations to assess for cataract development 3. Monthly pregnancy tests for all women of childbearing age 4. Complete blood count every 3 to 4 months

4. Complete blood count every 3 to 4 months

Establishing glycemic targets is the first step in treatment of both types of diabetes. For type 1 diabetes: 1. Tight control/intensive therapy can be given to adults who are willing to test their blood glucose at least twice daily. 2. Tight control is acceptable for older adults if they are without complications. 3. Plasma glucose levels are the same for children as adults. 4. Conventional therapy has a fasting plasma glucose target between 120 and 150 mg/dl.

4. Conventional therapy has a fasting plasma glucose target between 120 and 150 mg/dl.

Cecilia presents with depression associated with complaints of fatigue, sleeping all the time, and lack of motivation. An appropriate initial antidepressant for her would be: 1. Fluoxetine (Prozac) 2. Paroxetine (Paxil) 3. Amitriptyline (Elavil) 4. Duloxetine (Cymbalta)

4. Duloxetine (Cymbalta)

Levetiracetam has known drug interactions with: 1. Combined oral contraceptives 2. Carbamazepine 3. Warfarin 4. Few, if any, drugs

4. Few, if any, drugs

Drugs used to treat diabetic peripheral neuropathy include: 1. Metoclopramide 2. Cholinergic agonists 3. Cardioselective beta blockers 4. Gabapentin

4. Gabapentin

The tricyclic antidepressants should be prescribed cautiously in patients with: 1. Eczema 2. Asthma 3. Diabetes 4. Heart disease

4. Heart disease

Jack, age 8, has attention deficit disorder (ADD) and is prescribed methylphenidate (Ritalin). He and his parents should be educated about the side effects of methylphenidate, which are: 1. Slurred speech and insomnia 2. Bradycardia and confusion 3. Dizziness and orthostatic hypotension 4. Insomnia and decreased appetite

4. Insomnia and decreased appetite

Nonselective beta blockers and alcohol create serious drug interactions with insulin because they: 1. Increase blood glucose levels 2. Produce unexplained diaphoresis 3. Interfere with the ability of the body to metabolize glucose 4. Mask the signs and symptoms of altered glucose levels

4. Mask the signs and symptoms of altered glucose levels

The chemicals that promote the spread of pain locally include: 1. Serotonin 2. Norepinephrine 3. Enkephalin 4. Neurokinin A

4. Neurokinin A

If interventions to resolve the cause of pain (e.g., rest, ice, compression, and elevation) are insufficient, pain medications are given based on the severity of pain. Drugs are given in which order of use? 1. NSAIDs, opiates, corticosteroids 2. Low-dose opiates, salicylates, increased dose of opiates 3. Opiates, non-opiates, increased dose of non-opiate 4. Non-opiate, increased dose of non-opiate, opiate

4. Non-opiate, increased dose of non-opiate, opiate

Selma, who is overweight, recently started taking topiramate for seizures and at her follow-up visit you note she has lost 3 kg. The appropriate action would be: 1. Tell her to increase her caloric intake to counter the effects of the topiramate. 2. Consult with a neurologist, as this is not a common adverse effect of topiramate. 3. Decrease her dose of topiramate. 4. Reassure her that this is a normal side effect of topiramate and continue to monitor her weight.

4. Reassure her that this is a normal side effect of topiramate and continue to monitor her weight.

Monitoring of a patient on gabapentin to treat seizures includes: 1. Routine therapeutic drug levels every 3 to 4 months 2. Assessing for dermatologic reactions, including Steven's Johnson 3. Routine serum electrolytes, especially in hot weather 4. Recording seizure frequency, duration, and severity

4. Recording seizure frequency, duration, and severity

Protein restriction helps slow the progression of albuminuria, glomerular filtration rate, decline, and end stage renal disease in some patients with diabetes. It is useful for patients who: 1. Cannot tolerate angiotensin converting enzyme inhibitors or angiotensin receptor blockers 2. Have uncontrolled hypertension 3. Have HbA1C levels above 7% 4. Show progression of diabetic nephropathy despite optimal glucose and blood pressure control

4. Show progression of diabetic nephropathy despite optimal glucose and blood pressure control

Besides cystic fibrosis, which other medical state may trigger the need for pancreatic enzymes? 1. Paget's disease 2. Pulmonary cancers 3. Gallbladder surgery 4. Some bariatric surgeries

4. Some bariatric surgeries

Diagnostic criteria for diabetes include: 1. Fasting blood glucose greater than 140 mg/dl on two occasions 2. Postprandial blood glucose greater than 140 mg/dl 3. Fasting blood glucose 100 to 125 mg/dl on two occasions 4. Symptoms of diabetes plus a casual blood glucose greater than 200 mg/dl

4. Symptoms of diabetes plus a casual blood glucose greater than 200 mg/dl

Paige has a history of chronic migraines and would benefit from preventative medication. Education regarding migraine preventive medication includes: 1. Medication is taken at the beginning of the headache to prevent it from getting worse. 2. Medication alone is the best preventative against migraines occurring. 3. Medication should not be used more than four times a month. 4. The goal of treatment is to reduce migraine occurrence by 50%.

4. The goal of treatment is to reduce migraine occurrence by 50%.

Paige has a history of chronic migraines and would benefit from preventative medication. Education regarding migraine preventive medication includes: 1. Medication is taken at the beginning of the headache to prevent it from getting worse. 2. Medication alone is the best preventative against migraines occurring. 3. Medication should not be used more than four times a month. 4. The goal of treatment is to reduce migraine occurrence by 50%.

4. The goal of treatment is to reduce migraine occurrence by 50%.

The laboratory monitoring required when a patient is on a selective serotonin reuptake inhibitor is: 1. Complete blood count every 3 to 4 months 2. Therapeutic blood levels every 6 months after a steady state is achieved 3. Blood glucose every 3 to 4 months 4. There is no laboratory monitoring required

4. There is no laboratory monitoring required

Common mistakes practitioners make in treating anxiety disorders include: 1. Switching medications after an 8- to 12-week trial 2. Maximizing dosing of antianxiety medications 3. Encouraging exercise and relaxation therapy before starting medication 4. Thinking a partial response to medication is acceptable

4. Thinking a partial response to medication is acceptable

The drugs recommended for older adults with type 2 diabetes include: 1. Second-generation sulfonylureas 2. Metformin 3. Pioglitazone 4. Third-generation sulfonylureas

4. Third-generation sulfonylureas

Patients prescribed aspirin therapy require education regarding the signs of aspirin toxicity. An early sign of aspirin toxicity is: 1. Black tarry stools 2. Vomiting 3. Tremors 4. Tinnitus

4. Tinnitus

Cynthia is taking valproate (Depakote) for seizures and would like to get pregnant. What advice would you give her? 1. Valproate is safe during all trimesters of pregnancy. 2. She can get pregnant while taking valproate, but she should take adequate folic acid. 3. Valproate is not safe at any time during pregnancy. 4. Valproate is a known teratogen, but may be taken after the first trimester if necessary.

4. Valproate is a known teratogen, but may be taken after the first trimester if necessary.

When prescribing NSAIDS, a complete drug history should be conducted as NSAIDs interact with these drugs: 1. Omeprazole, a proton pump inhibitor 2. Combined oral contraceptives 3. Diphenhydramine, an antihistamine 4. Warfarin, an anticoagulant

4. Warfarin, an anticoagulant

loperamide dosing

4mg 4 times a day

Emollient laxative dosing

50-360 mg in divided doses

Biscodyl (diphenylmethane) & Phenolphthalein (Diphenymathane) onset

6-10 hrs

simethicone doses

60-180 mg four times a day

PPI last for about

72 hours after dose

After H. pylori treatment is completed, the next step in peptic ulcer disease therapy is:

A proton pump inhibitor for 8 to 12 weeks until healing is complete

Type 2 diabetes is a complex disorder involving

A suboptimal response of insulin-sensitive tissues in the liver.

Jamison has been prescribed citalopram (Celexa) to treat his depression. Education regarding how quickly SSRI antidepressants work would be: A. Appetite and concentration improve in the first 1 to 2 weeks B. Sleep should improve almost immediately upon starting citalopram C. Full response to the SSRI may take 2 to 4 months after he reaches full therapeutic dose D. His dysphoric mood will improve in 1 to 2 weeks

A. Appetite and concentration improve in the first 1 to 2 weeks

Rabi is being prescribed phenytoin for seizures. Monitoring includes: A. Assessing for phenytoin hypersensitivity syndrome 3 to 8 weeks after starting treatment B. Assessing for pedal edema throughout therapy C. Assessing heart rate at each visit and consider altering therapy if heart rate is less than 60 bpm D. Assessing for vision changes, such as red-green blindness, at least annually

A. Assessing for phenytoin hypersensitivity syndrome 3 to 8 weeks after starting treatment

When prescribing Adderall (amphetamine and dextroamphetamine) to adults with ADHD the nurse practitioner will need to monitor: A. Blood pressure B. Blood glucose levels C. Urine ketone levels D. Liver function

A. Blood pressure

A 19-year-old male was started on risperidone. Monitoring for risperidone includes observing for common side effects, including: A. Bradykinesia, akathisia, and agitation B. Excessive weight gain C. Hypertension D. Potentially fatal agranulocytosis

A. Bradykinesia, akathisia, and agitation

Taylor is a 10 year old diagnosed with major depression. The appropriate first line antidepressant for children is: A. Fluoxetine B. Fluvoxamine C. Sertraline D. Escitalopram

A. Fluoxetine

Prior to starting antidepressants, patients should have laboratory testing to rule out: A. Hypothyroidism B. Anemia C. Diabetes mellitus D. Low estrogen levels

A. Hypothyroidism

Kirk sprained his ankle and is asking for pain medication for his mild-to-moderate pain. The appropriate first line medication would be: A. Ibuprofen (Advil) B. Acetaminophen with hydrocodone (Vicodin) C. Oxycodone (Oxycontin) D. Oral morphine (Roxanol)

A. Ibuprofen (Advil)

Patients who are on chronic long-term proton pump inhibitor therapy require monitoring for: A. Iron deficiency anemia, vitamin B12 and calcium deficiency B. Folate and magnesium deficiency C. Elevated uric acid levels leading to gout D. Hypokalemia and hypocalcemia

A. Iron deficiency anemia, vitamin B12 and calcium deficiency

Patrick is a 10 year old who presents with constipation. Along with diet changes, a laxative is ordered to provide more rapid relief of constipation. An appropriate choice of medication for a 10 year old would be: A. PEG 3350 (Miralax) B. Bisacodyl (Dulcolax) suppository C. Docusate (Colace) suppository D. Polyethylene glycol electrolyte solution

A. PEG 3350 (Miralax)

Many patients self-medicate with antacids. Which patients should be counseled to not take calcium carbonate antacids without discussing with their provider or a pharmacist first? A. Patients with kidney stones B. Pregnant patients C. Patients with heartburn D. Post-menopausal women

A. Patients with kidney stones

Anticholinergic agents, such as benztropine (Cogentin), may be given with a phenothiazine to: A. Reduce the chance of tardive dyskinesia B. Potentiate the effects of the drug C. Reduce the tolerance which tends to occur D. Increase CNS depression

A. Reduce the chance of tardive dyskinesia

Suzanne is started on paroxetine (Paxil), an SSRI, for depression. Education regarding her antidepressant includes: A. SSRIs may take 2 to 6 weeks before she will have maximum drug effects B. Red-green color blindness may occur and should be reported C. If she experiences dry mouth or heart rates greater than 80, stop taking the drug immediately D. She should eat lots of food high in fiber to prevent constipation

A. SSRIs may take 2 to 6 weeks before she will have maximum drug effects

Cara is taking levetiracetam (Keppra) to treat seizures. Routine education for levetiracetam includes reminding her: A. To not abruptly discontinue levetiracetam due to risk for withdrawal seizures B. To wear sunscreen due to photosensitivity from levetiracetam C. To get an annual eye exam while on levetiracetam D. To report weight loss if it occurs

A. To not abruptly discontinue levetiracetam due to risk for withdrawal seizures

An appropriate first-line drug for the treatment of depression with fatigue and low energy would be: A. Venlafaxine (Effexor) B. Escitalopram (Lexapro) C. Buspirone (Buspar) D. Amitriptyline (Elavil)

A. Venlafaxine (Effexor)

Patients should be instructed regarding the rapid onset of zolpidem (Ambien) because: A. Zolpidem should be taken just before going to bed B. Zolpidem may cause dry mouth and constipation C. Patients may need to double the dose for effectiveness D. They should stop drinking alcohol at least 30 minutes before taking zolpidem

A. Zolpidem should be taken just before going to bed

Henry is eighty-two years old and takes two aspirin every morning to treat the arthritis pain in his back. He states that the aspirin helps him to "get going" each day. Lately, he has had some heartburn from the aspirin. After ruling out an acute GI bleed, what would be an appropriate course of treatment for Henry? A. Add an H2 blocker such as ranitidine to his therapy. B. Discontinue the aspirin and switch him to Vicodin for the pain. C. Decrease the aspirin dose to one tablet daily. D. Have Henry take an antacid fifteen minutes before taking the aspirin each day.

A. Add an H2 blocker such as ranitidine to his therapy.

The drug recommended as primary prevention of osteoporosis in men over seventy years is: A. Alendronate (Fosamax) B. Ibandronate (Boniva) C. Calcium carbonate D. Raloxifene (Evista)

A. Alendronate (Fosamax)

The drug recommended as primary prevention of osteoporosis in women over seventy years old is: A. Alendronate (Fosamax) B. Ibandronate (Boniva) C. Calcium carbonate D. Raloxifene (Evista)

A. Alendronate (Fosamax)

Judy is being prescribed phenytoin for seizures. Monitoring includes: A. Assessing for phenytoin hypersensitivity syndrome three to eight weeks after starting treatment B. Assessing for pedal edema throughout therapy C. Assessing the heart rate at each visit and consider altering therapy if the heart rate is less than 60 bpm D. Assessing for vision changes, such as red-green blindness, at least annually

A. Assessing for phenytoin hypersensitivity syndrome three to eight weeks after starting treatment

A twenty-two-year-old woman receives a prescription for oral contraceptives. Education for this patient includes: A. Counseling regarding decreasing or not smoking while taking oral contraceptives B. Advising a monthly pregnancy test for the first three months she is taking the contraceptive C. Advising that she may miss two pills in a row and not be concerned about pregnancy D. Informing her that her next follow-up visit is in one year for a refill and "annual exam"

A. Counseling regarding decreasing or not smoking while taking oral contraceptives

Prior to starting antidepressants, patients should have laboratory testing to rule out: A. Hypothyroidism B. Anemia C. Diabetes mellitus D. Low estrogen levels

A. Hypothyroidism

Bisphosphonates treat or prevent osteoporosis by: A. Inhibiting osteoclastic activity B. Fostering bone resorption C. Enhancing calcium uptake in bones D. Strengthening the osteoclastic proton pump

A. Inhibiting osteoclastic activity

Infants with congenital hypothyroidism are treated with: A. Levothyroxine B. Liothyronine C. Liotrix D. Methimazole

A. Levothyroxine

The Pain Management Contract is appropriate for which of the following cases? A. Patients with a history of chemical dependency or possible inappropriate use of pain medications B. All patients with chronic pain who will require long-term use of opiates C. Patients who have a complex drug regimen D. Patients who see multiple providers for pain control

A. Patients with a history of chemical dependency or possible inappropriate use of pain medications

Age is a factor in different responses to pain. Which of the following age-related statements about pain is not true? A. Preterm and newborn infants do not yet have functional pain pathways. B. Painful experiences and prolonged exposure to analgesic drugs during pregnancy may permanently alter neuronal organization in the child. C. Increases in pain threshold in older adults may be related to peripheral neuropathies and changes in skin thickness. D. Decreases in pain tolerance are evident in older adults.

A. Preterm and newborn infants do not yet have functional pain pathways.

When prescribing Adderall (amphetamine and dextroamphetamine) to adults with ADHD, the nurse practitioner will need to monitor: A. The blood pressure B. Blood glucose levels C. Urine ketone levels D. Liver function

A. The blood pressure

Men who are prescribed sildenafil (Viagra) need ongoing monitoring for: A. The development of chest pain or dizziness B. Weight gain C. Priapism D. Renal function

A. The development of chest pain or dizziness

Cara is taking levetiracetam (Keppra) to treat seizures. Routine education for levetiracetam includes reminding her: A. To not abruptly discontinue levetiracetam due to the risk of withdrawal seizures B. To wear a sunscreen due to photosensitivity from levetiracetam C. To get an annual eye exam while on levetiracetam D. To report weight loss if it occurs

A. To not abruptly discontinue levetiracetam due to the risk of withdrawal seizures

An appropriate first-line drug for the treatment of depression with fatigue and low energy would be: A. Venlafaxine (Effexor) B. Escitalopram (Lexapro) C. Buspirone (Buspar) D. Amitriptyline (Elavil)

A. Venlafaxine (Effexor)

Patients should be instructed regarding the rapid onset of zolpidem (Ambien) because: A. Zolpidem should be taken just before going to bed. B. Zolpidem may cause a dry mouth and constipation. C. Patients may need to double the dose for effectiveness. D. Patients should stop drinking alcohol at least thirty minutes before taking zolpidem.

A. Zolpidem should be taken just before going to bed.

Kirk sprained his ankle and is asking for pain medication for his mild-to-moderate pain. The appropriate first-line medication would be __________. A. ibuprofen (Advil) B. acetaminophen with hydrocodone (Vicodin) C. oxycodone (OxyContin) D. oral morphine (Roxanol)

A. ibuprofen (Advil)

Henry is 82 years old and takes two aspirin every morning to treat the arthritis pain in his back. He states the aspirin helps him to "get going" each day. Lately he has had some heartburn from the aspirin. After ruling out an acute GI bleed, what would be an appropriate course of treatment for Henry?

Add an H2 blocker such as ranitidine to his therapy.

Infants with reflux are initially treated with:

Anti-reflux maneuvers (elevate head of bed)

Treatment of IBS

Anticholinergic agents (Dicyclomine and hyoscyamine) & antidepressants (tricyclic antidepressants, SSRI)

Treatment failure in patients with peptic ulcer disease associated with H. pylori may be because of:

Antimicrobial resistance

Narcotics are exogenous opiates. They act by:

Attaching to receptors in the afferent neuron to inhibit the release of substance P

Type 1 diabetes results from autoimmune destruction of the beta cells. Eighty-five to 90 percent of type 1 diabetics have:

Auto antibodies to two tyrosine phosphatases

Kasey fractured his ankle in two places and is asking for pain medication for his pain. The appropriate first line medication would be: A. Ibuprofen (Advil) B. Acetaminophen with hydrocodone (Vicodin) C. Oxycodone (Oxycontin) D. Oral morphine ((Roxanol)

B. Acetaminophen with hydrocodone (Vicodin)

Sarah, a 42-year-old female, requests a prescription for an anorexiant to treat her obesity. A trial of phentermine is prescribed. Prescribing precautions include: A. Understanding that obesity is a contraindication to prescribing phentermine B. Anorexiants may cause tolerance and should only be prescribed for 6 months C. Patients should be monitored for postural hypotension D. Renal function should be monitored closely while on anorexiants

B. Anorexiants may cause tolerance and should only be prescribed for 6 months

Sook has been prescribed gabapentin to treat neuropathic pain and is complaining of feeling depressed and having "strange" thoughts. The appropriate initial action would be: A. Increase her dose B. Assess for suicidal ideation C. Discontinue the medication immediately D. Decrease her dose to half then slowly titrate up the dose

B. Assess for suicidal ideation

Hannah will be traveling to Mexico with her church group over Spring Break to build houses. She is concerned she may develop traveler's diarrhea. Advice includes normal food and water precautions as well as: A. Loperamide four times a day throughout the trip B. Bismuth subsalicylate with each meal and bedtime C. A prescription for diphenoxylate with atropine to use if she gets diarrhea D. None of the above

B. Bismuth subsalicylate with each meal and bedtime

Dwayne has recently started on carbamazepine to treat seizures. He comes to see you and you note that while his carbamazepine levels had been in the therapeutic range, they are now low. The possible cause for the low carbamazepine levels include: A. Dwayne hasn't been taking his carbamazepine because it causes insomnia B. Carbamazepine auto-induces metabolism, leading to lower levels in spite of good compliance C. Dwayne was not originally prescribed the correct amount of carbamazepine D. Carbamazepine is probably not the right antiseizure medication for Dwayne

B. Carbamazepine auto-induces metabolism, leading to lower levels in spite of good compliance

An appropriate drug to initially treat panic disorder is: A. Alprazolam (Xanax) B. Diazepam (Valium) C. Buspirone (Buspar) D. Amitriptyline (Elavil)

B. Diazepam (Valium)

Tom is taking lithium for bipolar disorder. He should be taught to: A. Take his lithium with food B. Eat a diet with consistent levels of salt (sodium) C. Drink at least 2 quarts of water if he is in a hot environment D. Monitor blood glucose levels

B. Eat a diet with consistent levels of salt (sodium)

An appropriate drug for the treatment of depression with anxiety would be: A. Alprazolam (Xanax) B. Escitalopram (Lexapro) C. Buspirone (Buspar) D. Amitriptyline (Elavil)

B. Escitalopram (Lexapro)

David is a 34 year old who is starting on paroxetine (Paxil) for depression. David's education regarding his medication would include: A. Paroxetine may cause intermittent diarrhea B. He may experience sexual dysfunction beginning a month after he starts therapy C. He may have constipation and he should increase fluids and fiber D. Paroxetine has a long half-life so he may occasionally skip a dose

B. He may experience sexual dysfunction beginning a month after he starts therapy

Scott's seizures are well controlled on topiramate and he wants to start playing baseball. Education for Scott regarding his topiramate includes: A. He should not play sports due to the risk of increased seizures B. He should monitor his temperature and ability to sweat in the heat while playing C. Reminding him that he may need higher dosages of topiramate when exercising D. Encouraging him to use sunscreen due to photosensitivity from topiramate

B. He should monitor his temperature and ability to sweat in the heat while playing

Sadie is a 72 year old who takes omeprazole for her chronic GERD. Chronic long-term omeprazole use places her at increased risk for: A. Megaloblastic anemia B. Osteoporosis C. Hypertension D. Strokes

B. Osteoporosis

Jim presents with complaints of "heart burn" that is minimally relieved with Tums (calcium carbonate) and is diagnosed with GERD. An appropriate first step therapy would be: A. Omeprazole (Prilosec) twice a day B. Ranitidine (Zantac) twice a day C. Famotidine (Pepcid) once a day D. Metoclopramide (Reglan) four times a day

B. Ranitidine (Zantac) twice a day

Antonia is a 3 year old who has a history of status epilepticus. Along with her routine antiseizure medication, she should also have a home prescription for____ to be used for an episode of status epilepticus. A. IV phenobarbital B. Rectal diazepam (Diastat) C. IV phenytoin (Dilantin) D. Oral carbamazepine (Tegretol)

B. Rectal diazepam (Diastat)

Jaycee has been on escitalopram (Lexapro) for a year and is willing to try tapering off of the SSRI. What is the initial dosage adjustment when starting a taper off antidepressants? A. Change dose to every other day dosing for a week B. Reduce dose by 50% for 3 to 4 days C. Reduce dose by 50% every other day D. Escitalopram (Lexapro) can be stopped abruptly due to its long half-life

B. Reduce dose by 50% for 3 to 4 days

Zainab is taking lamotrigine (Lamictal) and presents to the clinic with fever and lymphadenopathy. Initial evaluation and treatment includes: A. Reassuring her she has a viral infection and to call if she isn't better in 4 or 5 days B. Ruling out a hypersensitivity reaction that may lead to multi-organ failure C. Rapid strep test and symptomatic care if strep test is negative D. Observation only, with further assessment if she worsens

B. Ruling out a hypersensitivity reaction that may lead to multi-organ failure

Kelly has diarrhea and is wondering if she can take loperamide (Imodium) for the diarrhea. Loperamide: A. Can be given to all age patients, including infants and children, for viral gastroenteritis B. Slows gastric motility and reduces fluid and electrolyte loss from diarrhea C. Is the treatment of choice for the diarrhea associated with E. coli 0157 D. May be used in pregnancy and by lactating women

B. Slows gastric motility and reduces fluid and electrolyte loss from diarrhea

When prescribing temazepam (Restoril) for insomnia, patient education includes: A. Take temazepam nightly approximately 15 minutes before bedtime B. Temazepam should not be used more than three times a week for less than 3 months C. Drinking 1 ounce of alcohol will cause additive effects and the patient will sleep better D. Exercise for at least 30 minutes within 2 hours of bedtime to enhance the effects of temazepam

B. Temazepam should not be used more than three times a week for less than 3 months

atients who are prescribed olanzapine (Zyprexa) should be monitored for: A. Insomnia B. Weight gain C. Hypertension D. Galactorrhea

B. Weight gain

Jake, a 45-year-old patient with schizophrenia, was recently hospitalized for acute psychosis due to medication noncompliance. He was treated with IM long-acting haloperidol. Besides monitoring his schizophrenia symptoms, the patient should be assessed by his primary care provider: A. For excessive weight loss B. With the Abnormal Involuntary Movement Scale (AIMS) for EPS symptoms C. Monthly for tolerance to the haloperidol D. Only by the mental health provider, as most NPs in primary care do not care for mentally ill patients

B. With the Abnormal Involuntary Movement Scale (AIMS) for EPS symptoms

Vicky, age fifty-six years, comes to clinic requesting a refill of her Fiorinal (aspirin and butalbital) that she takes for migraines. She has been taking this medication for over two years for migraine and states one dose usually works to abort her migraine. What is the best care for her? A. Switch her to sumatriptan (Imitrex) to treat her migraines. B. Assess how often she is using Fiorinal and refill medication. C. Switch her to a beta-blocker such as propranolol to prevent her migraine. D. Request her to return to the original prescriber of Fiorinal as you do not prescribe butalbital for migraines.

B. Assess how often she is using Fiorinal and refill medication.

Angela is a black woman who has heard that women of African descent do not need to worry about osteoporosis. What education would you provide Angela about her risk? A. She is correct; black women do not have much risk of developing osteoporosis due to their dark skin. B. Black women are at risk of developing osteoporosis due to their lower calcium intake as a group. C. If she doesn't drink alcohol, her risk of developing osteoporosis is low. D. If she has not lost more than 10% of her weight lately, her risk is low.

B. Black women are at risk of developing osteoporosis due to their lower calcium intake as a group.

Absolute contraindications to estrogen therapy include: A. A history of any type of cancer B. Clotting disorders C. A history of tension headache D. Orthostatic hypotension

B. Clotting disorders

Shana is receiving her first medroxyprogesterone (Depo Provera) injection. Shana will need to be monitored for: A. Depression B. Hypertension C. Weight loss D. Cataracts

B. Hypertension

Henry presents to clinic with a significantly swollen, painful great toe and is diagnosed with gout. Of the following, which would be the best treatment for Henry? A. High-dose colchicines B. Low-dose colchicines C. High-dose aspirin D. Acetaminophen with codeine

B. Low-dose colchicines

A woman who is pregnant and has hyperthyroidism is best managed by a specialty team that will most likely treat her with: A. Methimazole. B. Propylthiouracil. C. Radioactive iodine. D. Nothing; treatment is best delayed until after her pregnancy ends.

B. Propylthiouracil.

Jaycee has been on escitalopram (Lexapro) for a year and is willing to try tapering off of the selective-serotonin reuptake inhibitors. What is the initial dosage adjustment when starting a taper off antidepressants? A. Change the dose to every other day dosing for a week. B. Reduce the dose by 50% for three to four days. C. Reduce the dose by 50% every other day. D. Escitalopram (Lexapro) can be stopped abruptly due to its long half-life

B. Reduce the dose by 50% for three to four days

A woman who has migraine with aura: A. Should not be prescribed estrogen due to the interaction between triptans and estrogen, limiting migraine therapy choices B. Should not be prescribed estrogen due to an increased incidence of migraines with the use of estrogen C. Should not be prescribed estrogen due to an increased risk of stroke occurring with estrogen use D. May be prescribed estrogen without any concerns

B. Should not be prescribed estrogen due to an increased risk of stroke occurring with estrogen use

Elderly patients who are started on levothyroxine for thyroid replacement should be monitored for: A. Excessive sedation B. Tachycardia and angina C. Weight gain D. Cold intolerance

B. Tachycardia and angina

Patients who are prescribed olanzapine (Zyprexa) should be monitored for: A. Insomnia B. Weight gain C. Hypertension D. Galactorrhea

B. Weight gain

Jake, a forty-five-year-old patient with schizophrenia, was recently hospitalized for acute psychosis due to medication noncompliance. He was treated with intramuscular (IM) long-acting haloperidol. Besides being monitored for his schizophrenia symptoms, the patient should be assessed by his primary care provider: A. For excessive weight loss B. With the Abnormal Involuntary Movement Scale (AIMS) for extrapyramidal symptoms (EPS) symptoms C. Monthly for tolerance to the haloperidol D. Only by the mental health provider as most nurse practitioners in primary care do not care for mentally ill patients

B. With the Abnormal Involuntary Movement Scale (AIMS) for extrapyramidal symptoms (EPS) symptoms

Narcotics are exogenous opiates. They act by ______. A. inhibiting pain transmission in the spinal cord B. attaching to receptors in the afferent neuron to inhibit the release of substance P C. blocking neurotransmitters in the midbrain D. increasing beta-lipoprotein excretion from the pituitary

B. attaching to receptors in the afferent neuron to inhibit the release of substance P

Juanita presents to clinic with a complaint of headaches off and on for months. She reports she feels like someone is "squeezing" her head. She occasionally takes Tylenol for the pain but usually just "toughs it out." Initial treatment for tension headache includes asking her to keep a headache diary and a prescription for ________. A. sumatriptan (Imitrex) B. naproxen (Aleve) C. ergotamine (Ergostat) D. Tylenol with codeine (Tylenol #3)

B. naproxen (Aleve)

Patrick is a 10-year-old patient who presents with uncomfortable constipation. Along with diet changes, a laxative is ordered to provide more rapid relief of constipation. An appropriate choice of medication for a 10-year-old child would be:

Bisacodyl (Dulcolax) suppository

Stimulant laxatives that can be used only as acute treatment

Biscodyl (diphenylmethane) Phenolphthalein (Diphenymathane), Caster Oil

Hannah will be traveling to Mexico with her church group over spring break to build houses. She is concerned she may develop traveler's diarrhea. Advice includes following normal food and water precautions as well as taking:

Bismuth subsalicylate with each meal and at bedtime

Control targets for patients with diabetes include

Blood pressure less than 130/80mm Hg

In choosing a benzodiazepam to treat anxiety the prescriber needs to be aware of the possibility of dependence. The benzodiazepam with the greatest likelihood of rapidly developing dependence is: A. Chlordiazepoxide (Librium) B. Clonazepam (Klonopin) C. Alprazolam (Xanax) D. Oxazepam (Serax)

C. Alprazolam (Xanax)

Six-year-old Lucy has recently been started on ethosuximide (Zarontin) for seizures. She should be monitored for: A. Increased seizure activity, as this drug may auto-induce seizures B. Altered renal function, including renal failure C. Blood dyscrasias, which are uncommon but possible D. Central nervous system excitement, leading to insomnia

C. Blood dyscrasias, which are uncommon but possible

An appropriate first-line drug to try for mild to moderate generalized anxiety disorder would be: A. Alprazolam (Xanax) B. Diazepam (Valium) C. Buspirone (Buspar) D. Amitriptyline (Elavil)

C. Buspirone (Buspar)

Carbamazepine has a Black Box warning due to life-threatening: A. Renal toxicity, leading to renal failure B. Hepatotoxicity, leading to liver failure C. Dermatologic reaction, including Steven's Johnson and toxic epidermal necrolysis D. Cardiac effects, including supraventricular tachycardia

C. Dermatologic reaction, including Steven's Johnson and toxic epidermal necrolysis

One major drug used to treat bipolar disease is lithium. Because lithium has a narrow therapeutic range, it is important to recognize symptoms of toxicity, such as: A. Orthostatic hypotension B. Agitation and irritability C. Drowsiness and nausea D. Painful urination and abdominal distention

C. Drowsiness and nausea

Samantha is taking lamotrigine (Lamictal) for her seizures and requests a prescription for oral contraceptives. Combined oral contraceptives (OCs) interact with lamotrigine and may cause: A. Contraceptive failure B. Excessive weight gain C. Reduced lamotrigine levels, requiring doubling the dose of lamotrigine D. Induction of estrogen metabolism, requiring higher estrogen content OCs be prescribed

C. Reduced lamotrigine levels, requiring doubling the dose of lamotrigine

Before prescribing phentermine to Sarah, a thorough drug history should be taken including assessing for the use of serotonergic agents such as SSRIs and St John's Wort due to: A. Additive respiratory depression risk B. Additive effects affecting liver function C. The risk of serotonin syndrome D. The risk of altered cognitive functioning

C. The risk of serotonin syndrome

In choosing a benzodiazepam to treat anxiety, the prescriber needs to be aware of the possibility of dependence. The benzodiazepam with the greatest likelihood of rapidly developing dependence is: A. Chlordiazepoxide (Librium) B. Clonazepam (Klonopin) C. Alprazolam (Xanax) D. Oxazepam (Serax)

C. Alprazolam (Xanax)

Inadequate vitamin D intake can contribute to the development of osteoporosis by: A. Increasing calcitonin production B. Increasing calcium absorption from the intestine C. Altering calcium metabolism D. Stimulating bone formation

C. Altering calcium metabolism

The ongoing monitoring of patients over the age sixty-five years taking alendronate (Fosamax) or any other bisphosphonate is: A. Annual dual energy X-ray absorptiometry (DEXA) scans B. Annual vitamin D level C. Annual renal function evaluation D. Electrolytes every three months

C. Annual renal function evaluation

Six-year-old Lucy has recently been started on ethosuximide (Zarontin) for seizures. She should be monitored for: A. Increased seizure activity as this drug may auto-induce seizures B. Altered renal function, including renal failure C. Blood dyscrasias, which are uncommon but possible D. CNS excitement, leading to insomnia

C. Blood dyscrasias, which are uncommon but possible

Prophylactic use of bisphosphonates is recommended for patients with early osteopenia related to long-term use of which of the following drugs? A. Selective estrogen-receptor modulators B. Aspirin C. Glucocorticoids D. Calcium supplements

C. Glucocorticoids

When starting a patient with hypothyroidism on thyroid replacement hormones, patient education would include the following: A. He or she should feel symptomatic improvement in one to two weeks. B. Drug-related adverse effects such as lethargy and dry skin may occur. C. It may take four to eight weeks to get to euthyroid symptomatically and by lab testing. D. Due to the short half-life of levothyroxine, its doses should not be missed.

C. It may take four to eight weeks to get to euthyroid symptomatically and by lab testing.

Which of the following is the goal of treatment of acute pain? A. Pain at a tolerable level where patient may return to activities of daily living B. Reduction of pain with a minimum of drug adverse effects C. Reduction or elimination of pain with minimum adverse reactions D. Adequate pain relief without constipation or nausea from the drugs

C. Reduction or elimination of pain with minimum adverse reactions

When prescribing any headache therapy, appropriate use of medications needs to be discussed to prevent medication-overuse headaches. The clinical characteristics of medication-overuse headaches include ________. A. headaches increasing in frequency B. headaches increasing in intensity C. headaches recurring when medication wears off D. headaches beginning to "cluster" into a pattern

C. headaches recurring when medication wears off

Sallie has been taking 10 mg of prednisone per day for the past six months. She should be assessed for ________. A. gout B. iron deficiency anemia C. osteoporosis D. renal dysfunction

C. osteoporosis

Chronic pain is a complex problem. Some specific strategies to deal with it include ________. A. telling the patient to "let pain be your guide" to using treatment therapies B. prescribing pain medication on a pro re nata (PRN) basis to keep down the amount used C. scheduling return visits on a regular basis rather than waiting for poor pain control to drive the need for an appointment D. All the given options

C. scheduling return visits on a regular basis rather than waiting for poor pain control to drive the need for an appointment

Dosage changes of conjugated equine estrogen (Premarin) are made at ____ intervals. A. one to two week B. two to four week C. six to eight week D. twelve week

C. six to eight week

Different areas of the brain are involved in specific aspects of pain. The reticular and limbic systems in the brain influence ______. A. the sensory aspects of pain B. the discriminative aspects of pain C. the motivational aspects of pain D. the cognitive aspects of pain

C. the motivational aspects of pain

Patients who are on long-term aspirin therapy should have ______ annually.

CBC

Lifestyle changes are the first step in treatment of gastroesophageal reflux disease (GERD). Food or drink that may aggravate GERD include:

Caffeine

Establishing glycemic target is the first step in treatment of both types of diabetes . For Type 1 diabetes

Conventional therapy has a fasting plasma glucose target between 120 and 150 mg/dl

A 66-year-old male was prescribed phenelzine (Nardil) while in an acute psychiatric unit for recalcitrant depression. The NP managing his primary health care needs to understand the following regarding phenelzine and other MAOIs: A. He should not be prescribed any serotonergic drug such as sumatriptan (Imitrex) B. MAOIs interact with many common foods, including yogurt, sour cream, and soy sauce C. Symptoms of hypertensive crisis (headache, tachycardia, sweating) require immediate treatment

D. All of the above

Bismuth subsalicylate (Pepto Bismol) is a common OTC remedy for gastrointestinal complaints. Bismuth subsalicylate: A. May lead to toxicity if taken with aspirin B. Is contraindicated in children with flu-like illness C. Has antimicrobial effects against bacterial and viral enteropathogens D. All of the above

D. All of the above

Monitoring for a child on methylphenidate for ADHD includes: A. ADHD symptoms B. Routine height and weight checks C. Amount of methylphenidate being used D. All of the above

D. All of the above

Patients taking antacids should be educated regarding these drugs, including: A. They may cause constipation or diarrhea B. Many are high in sodium C. Separate antacids from other medications by 1 hour D. All of the above

D. All of the above

When prescribing an opioid analgesic such as acetaminophen and codeine (Tylenol #3), instructions to the patient should include: A. The medication may cause sedation and they should not drive B. Constipation is a common side effect and they should increase fluids and fiber C. Patients should not take any other acetaminophen-containing medications at the same time D. All of the above

D. All of the above

A patient with anxiety and depression may respond to: A. Duloxetine (Cymbalta) B. Fluoxetine (Prozac) C. Oxazepam (Serax) D. Buspirone (Buspar) and a SSRI combined

D. Buspirone (Buspar) and a SSRI combined

Long-term monitoring of patients who are taking carbamazepine includes: A. Routine troponin levels to assess for cardiac damage B. Annual eye examinations to assess for cataract development C. Monthly pregnancy tests for all women of childbearing age D. Complete blood count every 3 to 4 months

D. Complete blood count every 3 to 4 months

Cecilia presents with depression associated with complaints of fatigue, sleeping all the time, and lack of motivation. An appropriate initial antidepressant for her would be: A. Fluoxetine (Prozac) B. Paroxetine (Paxil) C. Amitriptyline (Elavil) D. Duloxetine (Cymbalta)

D. Duloxetine (Cymbalta)

Levetiracetam has known drug interactions with: A. Oral contraceptives B. Carbamazepine C. Warfarin D. Few, if any, drugs

D. Few, if any, drugs

The tricyclic antidepressants should be prescribed cautiously in patients with: A. Eczema B. Asthma C. Diabetes D. Heart disease

D. Heart disease

Jack, age 8, has attention deficit disorder (ADD) and is prescribed methylphenidate (Ritalin). He and his parents should be educated about the side effects of methylphenidate, which are: A. Slurred speech and insomnia B. Bradycardia and confusion C. Dizziness and orthostatic hypotension D. Insomnia and decreased appetite

D. Insomnia and decreased appetite

Josie is a 5 year old who presents to the clinic with a 48-hour history of nausea, vomiting, and some diarrhea. She is unable to keep fluids down and her weight is 4 pounds less than her last recorded weight. Besides IV fluids, her exam warrants the use of an antinausea medication. Which of the following would be the appropriate drug to order for Josie? A. Prochlorperazine (Compazine) B. Meclizine (Antivert) C. Promethazine (Phenergan) D. Ondansetron (Zofran)

D. Ondansetron (Zofran)

Selma, who is overweight, recently started taking topiramate for seizures and at her follow-up visit you note she has lost 3 kg. The appropriate action would be: A. Tell her to increase her caloric intake to counter the effects of the topiramate B. Consult with a neurologist, as this is not a common adverse effect of topiramate C. Decrease her dose of topiramate D. Reassure her that this is a normal side effect of topiramate and continue to monitor her weight

D. Reassure her that this is a normal side effect of topiramate and continue to monitor her weight

Monitoring of a patient on gabapentin to treat seizures includes: A. Routine therapeutic drug levels every 3 to 4 months B. Assessing for dermatologic reactions, including Steven's Johnson C. Routine serum electrolytes, especially in hot weather D. Recording seizure frequency, duration, and severity

D. Recording seizure frequency, duration, and severity

The laboratory monitoring required when a patient is on an SSRI is: A. Complete blood count every 3 to 4 months B. Therapeutic blood levels every 6 months after steady state is achieved C. Blood glucose every 3 to 4 months D. There is no laboratory monitoring required

D. There is no laboratory monitoring required

Common mistakes practitioners make in treating anxiety disorders include: A. Switching medications after an 8- to 12-week trial B. Maximizing dosing of anti-anxiety medications C. Encouraging exercise and relaxation therapy before starting medication D. Thinking a partial response to medication is acceptable

D. Thinking a partial response to medication is acceptable

Cynthia is taking valproate (Depakote) for seizures and would like to get pregnant. What advice would you give her? A. Valproate is safe during all trimesters of pregnancy. B. She can get pregnant while taking valproate, but she should take adequate folic acid. C. Valproate is not safe at any time during pregnancy. D. Valproate is a known teratogen, but may be taken after the first trimester if necessary.

D. Valproate is a known teratogen, but may be taken after the first trimester if necessary.

Which of the following statements is true about age and pain? A. Use of drugs that depend heavily on the renal system for excretion may require dosage adjustments in very young children. B. Among the NSAIDs, indomethacin is the preferred drug because of lower adverse effects profiles than other NSAIDs. C. Older adults who have dementia probably do not experience much pain due to loss of pain receptors in the brain. D. Acetaminophen is especially useful in both children and adults because it has no effect on platelets and has fewer adverse effects than NSAIDs.

D. Acetaminophen is especially useful in both children and adults because it has no effect on platelets and has fewer adverse effects than NSAIDs.

Monitoring for a child on methylphenidate for ADHD includes: A. ADHD symptoms B. Routine height and weight checks C. Amount of methylphenidate being used D. All of the above

D. All of the above

Patients with RA who are on chronic low-dose prednisone will need co-treatment with which medications to prevent further adverse effects? A. A bisphosphonate B. Calcium supplementation C. Vitamin D D. All of the above

D. All of the above

A sixty-six-year-old male was prescribed phenelzine (Nardil) while in an acute psychiatric unit for recalcitrant depression. The nurse practitioner managing his primary healthcare needs to understand the following regarding phenelzine and other monoamine oxidase inhibitors (MAOIs): A. He should not be prescribed any serotonergic drug such as sumatriptan (Imitrex). B. MAOIs interact with many common foods, including yogurt, sour cream, and soy sauce. C. Symptoms of hypertensive crisis (headache, tachycardia, sweating, etc.) require immediate treatment. D. All the above options are correct.

D. All the above options are correct.

Pain assessment to determine adequacy of pain management is important for all patients. This assessment is done to determine which of the following options? A. If the diagnosis of source of pain is correct B. If the current regimen is adequate or different combinations of drugs and nondrug therapy are required C. If the patient is willing and able to be an active participant in his or her pain management D. All the given options

D. All the given options

Amber is a twenty-four-year-old who has had migraines for ten years. She reports a migraine on average of once a month. The migraines are effectively aborted with naratriptan (Amerge). When refilling Amber's naratriptan education would include which of the following? A. Naratriptan will interact with antidepressants, including selective serotonin reuptake inhibitors and St John's Wort, and she should inform any providers she sees that she has migraines. B. Continue to monitor her headaches; if the migraine is consistently happening around her menses, then there is preventive therapy available. C. Pregnancy is contraindicated when taking a triptan. D. All the given options are correct.

D. All the given options are correct.

When prescribed an opioid analgesic such as acetaminophen and codeine (Tylenol #3), which of the following instructions should the patient follow? A. The medication may cause sedation, so the patient should not drive. B. Constipation is a common side effect, so the patient should increase fluids and fiber. C. The patient should not take any other acetaminophen-containing medications at the same time. D. All the given options are correct.

D. All the given options are correct.

Cecilia presents with depression associated with complaints of fatigue, sleeping all the time, and lack of motivation. An appropriate initial antidepressant for her would be: A. Fluoxetine (Prozac) B. Paroxetine (Paxil) C. Amitriptyline (Elavil) D. Duloxetine (Cymbalta)

D. Duloxetine (Cymbalta)

Levetiracetam has known drug interactions with: A. Oral contraceptives B. Carbamazepine C. Warfarin D. Few, if any, drugs

D. Few, if any, drugs

The tricyclic antidepressants should be prescribed cautiously in patients with: A. Eczema B. Asthma C. Diabetes D. Heart disease

D. Heart disease

Jack, eight years old, has attention deficit disorder (ADD) and is prescribed methylphenidate (Ritalin). He and his parents should be educated about the side effects of methylphenidate, which are: A. Slurred speech and insomnia B. Bradycardia and confusion C. Dizziness and orthostatic hypotension D. Insomnia and decreased appetite

D. Insomnia and decreased appetite

Selective estrogen receptor modifiers (SERMs) treat osteoporosis by selectively: A. Inhibiting magnesium resorption in the kidneys B. Increasing calcium absorption from the gastrointestinal (GI) tract C. Acting on the bone to inhibit osteoblast activity D. Selectively acting on the estrogen receptors in the bone

D. Selectively acting on the estrogen receptors in the bone

Cynthia is taking valproate (Depakote) for seizures and would like to get pregnant. What advice would you give her? A. Valproate is safe during all trimesters of pregnancy. B. She can get pregnant while taking valproate, but she should take adequate folic acid. C. Valproate is not safe at any time during pregnancy. D. Valproate is a known teratogen but may be taken after the first trimester if necessary.

D. Valproate is a known teratogen but may be taken after the first trimester if necessary.

Daniel has been on 60 mg of prednisone for 10 days to treat a severe asthma exacerbation. It is time to discontinue the prednisone. How is prednisone discontinued?

Develop a tapering schedule to slowly wean Daniel off the prednisone.

First line for prevention of constipation

Docusate sodium (colase)... emollient laxitive

Before prescribing metformin , the provider should

Draw a serum creatinine level to assess renal function

An elderly person has been prescribed lactulose for treatment of chronic constipation. Monitoring with long-term treatment would include:

Electrolytes, including potassium and chloride

Gastroesophageal reflux disease may be aggravated by the following medication that affects lower esophageal sphincter (LES) tone:

Estrogen

Prevention of conversion from pre-diabetes in young children must take highest priority and should focus on:

Fostering LDL levels less than 100mg/dl and total cholesterol less than 170 mg/dl to prevent cardiovascular disease.

Drugs used to treat diabetic peripheral neuropathy include

Gabapentin

When the total daily insulin dose is split and given twice daily , which of the following rules may be followed

Give two thirds of the total dose in the morning and one-third in the evening

The American heart Association states that people with diabetes have 2 to 4 fold increase in the risk of dying from cardiovascular disease. Treatments and Targets that do not appear to decrease risk for micro- and macrovascular complications include

Glycemic targets between 7% and 7.5 %

All diabetic patients with hyperlipidemia should be treated with

HMG-Co A reductase inhibitors

Both ACE inhibitors and some Angiostensin - II receptor blockers have been approved in treating

HTN in diabetic and diabetic nephropathy

Sulfonylureas may be added to the treatment regimen for Type 2 diabetics when lifestyle modifications and metformin are insufficient to achieve target glucose levels. Sulfonylureas have been moved to Step 2 therapy because they:

Have significant risk for hypoglycemia

The American Diabetic Association has recommended which of the following tests for ongoing management of diabetes.

HbAIC

Adam has type 1 diabetes and plays tennis for his university . He exhibits a knowledge deficit about his insulin and his diagnosis. He should be taught that

He should be taught to increase his CHO intake during times of exercise.

Jamie has fractured his ankle and has received a prescription for acetaminophen and hydrocodone (Vicodin). Education when prescribing Vicodin includes:

He should not take any other acetaminophen-containing medications.

Phenolphthalein (Diphenymathane) may not give with

Hemhrroids; may turn urine pink or stool red

Long-term use of proton pump inhibitors may lead to:

Hip fractures in at-risk persons

Antacids treat gastroesophageal reflux disease by:

Increasing gastric pH

Metoclopramide improves gastroesophageal reflux disease symptoms by:

Increasing lower esophageal tone

Insulin is used to treat both types of diabetes. It acts by

Increasing peripheral glucose uptake by skeletal muscle and fat

One of the main drug classes used to treat acute pain is NSAIDs. They are used because:

Inflammation is a common cause of acute pain.

Insulin preparations are divided into categories based on onset , duration, and intensity of action following subcutaneous inject. Which of the following insulin preparations has the shortest onset and duration of action

Insulin glulisine

Unlike most Type 2 diabetics where obesity is a major issue. older adults with low body weight have higher risks for morbidity and mortality. The most reliable indicator of poor nutritional status in older adults is

Involuntary loss of 10% of the body weight in less than 6 months

Patients who are on chronic long-term proton pump inhibitor therapy require monitoring for:

Iron deficiency anemia, vitamin B12 and calcium deficiency

Josefina is a 2-year-old child with acute otitis media and an upper respiratory infection. Along with an antibiotic she receives a recommendation to treat the ear pain with ibuprofen. What education would her parent need regarding ibuprofen?

Josefina needs to be well-hydrated while taking ibuprofen.

Bismuth subsalicylate (Pepto Bismol) is a common OTC remedy for gastrointestinal complaints. Bismuth subsalicylate:

May lead to toxicity if taken with aspirin & Is contraindicated in children with flu-like illness & Has antimicrobial effects against bacterial and viral enteropathogens

Sadie is a 72-year-old patient who takes omeprazole for her chronic GERD. Chronic long-term omeprazole use places her at increased risk for:

Megaloblastic anemia

The drugs recommended by the American Academy of Pediatrics for use in children with diabetes (depending upon type of diabetes are:

Metformin and Insulin

Routine screening of asymptomatic adults for diabetes is appropriate for:

Native Americans, African Americans , and Hispanics

If interventions to resolve the cause of pain (e.g., rest, ice, compression, and elevation) are insufficient, pain medications are given based on the severity of pain. are given in which order of use?

Non-opiate, increased dose of non-opiate, opiate

Patients whose total dose of prednisone will exceed 1 gram will most likely need a second prescription for:

Omeprazole, a proton pump inhibitor to prevent peptic ulcer disease

Methylnaltrexone is used to treat constipation in:

Opioid-associated constipation

Dose range for mesalamine

PO or Rectal 4.8g/day

Patients need to be questioned about all pain sites because:

Patients tend to report the most severe or important in their perception.

Many patients self-medicate with antacids. Which patients should be counseled to not take calcium carbonate antacids without discussing it with their provider or a pharmacist first?

Patients with kidney stones

Secretory diarrhea

Persists when patient fasts

If a patient with symptoms of gastroesophageal reflux disease states that he has been self-treating at home with OTC (ranitidine) Histamine2 receptor antagonist daily, the appropriate treatment would be:

Proton pump inhibitor (omeprazole) for 12 weeks

An acceptable first-line treatment for peptic ulcer disease with positive H. pylori test is:

Proton pump inhibitor bid plus clarithromycin plus amoxicillin for 14 days

If a patient with H. pylori-positive peptic ulcer disease fails first-line therapy, the second-line treatment is:

Proton pump inhibitor bid plus metronidazole plus tetracycline plus bismuth subsalicylate for 14 days

When treating patients using the "Step-Down" approach the patient with gastroesophageal reflux disease is started on _______ first.

Proton pump inhibitors

Jim presents with complaints of "heartburn" that is minimally relieved with Tums (calcium carbonate) and is diagnosed with gastroesophageal reflux disease (GERD). An appropriate first-step therapy would be:

Ranitidine (Zantac) twice a day

The next step in treatment when a patient has been on proton pump inhibitors twice daily for 12 weeks and not improving is:

Referral for endoscopy

Sallie has been taking 10 mg per day of prednisone for the past 6 months. She should be assessed for:

Renal dysfunction

Patients who are on chronic long-term corticosteroid therapy need education regarding:

Reporting black tarry stools or abdominal pain

Diabetic autonomic neuropathy (DAN) is the earliest and most common complication of diabetes. Symptoms associated with DAN include

Resting tachycardia, exercise intolerance, and orthostatic hypotension

Patients who are on or who will be starting chronic corticosteroid therapy need monitoring of:

Serum glucose

Protein restriction helps slow the progression of albuminuria. GFR decline, and ESRD is some patients with diabetes . it is useful for patients who:

Show progression od diabetic nephropathy despite optimal glucose and blood pressure control.

Treatment with insulin for type 1 diabetes

Starts with a total daily of 0.2 to 0.4 units per kg of body weight.

Diagnostic criteria for diabetes include

Symptoms of diabetes plus a casual blood glucose greater than 200mg/dl

The drugs recommended for older adults with Type 2 diabetes include

Third generation sulfonylureas

Studies have shown that control targets that reduce the HbA1C to less than 7% are associated with fewer long-term complications of diabetes. Patients who should have such a target include:

Those with no significant cardiovascular disease

Patients prescribed aspirin therapy require education regarding the signs of aspirin toxicity. An early sign of aspirin toxicity is:

Tinnitus

If a patient with gastroesophageal reflux disease who is taking a proton pump inhibitor daily is not improving, the plan of care would be:

Use PPI TID for 4-8 weeks

When prescribing NSAIDS, a complete drug history should be conducted as NSAIDs interact with these drugs:

Warfarin, an anticoagulant

A 22-year-old woman receives a prescription for oral contraceptives. Education for this patient includes: Question options: Counseling regarding decreasing or not smoking while taking oral contraceptives Advising a monthly pregnancy test for the first 3 months she is taking the contraceptive Advising that she may miss two pills in a row and not be concerned about pregnancy Recommending that her next follow-up visit is in 1 year for a refill and annual exam

a

Kelly is a 14-year-old patient who presents to the clinic with a classic migraine. She says she is having a headache two to three times a month. The initial plan would be: Question options: Prescribe NSAIDs as abortive therapy and have her keep a headache diary to identify her triggers. Prescribe zolmitriptan (Zomig) as abortive therapy and recommend relaxation therapy to reduce her stress. Prescribe acetaminophen with codeine (Tylenol #3) for her to take at the first onset of her migraine. Prescribe sumatriptan (Imitrex) nasal spray and arrange for her to receive the first dose in the clinic.

a

Prior to prescribing metformin, the provider should: Question options: Draw a serum creatinine to assess renal function Try the patient on insulin Tell the patient to increase iodine intake Have the patient stop taking any sulfonylurea to avoid dangerous drug interactions

a

The American Heart Association states that people with diabetes have a 2- to 4-fold increase in the risk of dying from cardiovascular disease. Treatments and targets that do not appear to decrease risk for micro- and macro-vascular complications include: Question options: Glycemic targets between 7% and 7.5% Use of insulin in type 2 diabetics Control of hypertension and hyperlipidemia Stopping smoking

a

The drug recommended as primary prevention of osteoporosis in men over age 70 years is: Question options: Alendronate (Fosamax) Ibandronate (Boniva) Calcium carbonate Raloxifene (Evista)

a

The longer-term Xanax patient comes in and states they need a higher dose of the medication. They deny any additional, new, or accelerating triggers of their anxiety. What is the probable reason? Question options: They have become tolerant of the medication, which is characterized by the need for higher and higher doses. They are a drug seeker. They are suicidal. They only need additional counseling on lifestyle modification.

a

Treatment of a patient with hypothyroidism and cardiovascular disease consists of: Question options: Levothyroxine Liothyronine Liotrix Methimazole

a

Treatment with insulin for type 1 diabetics: Question options: Starts with a total daily dose of 0.2 to 0.4 units per kg of body weight Divides the total doses into three injections based on meal size Uses a total daily dose of insulin glargine given once daily with no other insulin required Is based on the level of blood glucose

a

Women with an intact uterus should be treated with both estrogen and progestin due to: Question options: Increased risk for endometrial cancer if estrogen alone is used Combination therapy provides the best relief of menopausal vasomotor symptoms Reduced risk for colon cancer with combined therapy Lower risk of developing blood clots with combined therapy

a

You are treating a patient with a diagnosis of Alzheimer's disease. The patient's wife mentions difficulty with transportation to the clinic. Which medication is the best choice? Question options: Donepezil Tacrine Doxazosin Verapamil

a

The action of "gliptins " is different from other antidiabetic agents because they

act on the incretin system to indirectly increase insulin production

First line for inflammatory bowel disease

aminosalicyclates (mesalamine, sulfasalazine) corticosteroids (budesonide, prednixone, hydrocortisone)

A first-line drug for abortive therapy in simple migraine is: Question options: Sumatriptan (Imitrex) Naproxen (Aleve) Butorphanol nasal spray (Stadol NS) Butalbital and acetaminophen (Fioricet)

b

An appropriate drug for the treatment of depression with anxiety would be: Question options: Alprazolam (Xanax) Escitalopram (Lexapro) Buspirone (Buspar) Amitriptyline (Elavil)

b

Antonia is a 3-year-old child who has a history of status epilepticus. Along with her routine antiseizure medication, she should also have a home prescription for_________ to be used for an episode of status epilepticus. Question options: IV phenobarbital Rectal diazepam (Diastat) IV phenytoin (Dilantin) Oral carbamazepine (Tegretol)

b

Bisphosphonate administration education includes: Question options: Taking it on a full stomach Requiring sitting erect for at least 30 minutes afterward Drinking it with orange juice Taking it with H2 blockers or proton pump inhibitors (PPI) to protect the stomach

b

Chemical dependency assessment is integral to the initial assessment of chronic pain. Which of the following raises a "red flag" about potential chemical dependency? Question options: Use of more than one drug to treat the pain Multiple times when prescriptions are lost with requests to refill Preferences for treatments that include alternative medicines Presence of a family member who has abused drugs

b

David is a 34-year-old patient who is starting on paroxetine (Paxil) for depression. David's education regarding his medication would include: Question options: Paroxetine may cause intermittent diarrhea. He may experience sexual dysfunction beginning a month after he starts therapy. He may have constipation and he should increase fluids and fiber. Paroxetine has a long half-life so he may occasionally skip a dose.

b

Dwayne has recently started on carbamazepine to treat seizures. He comes to see you and you note that while his carbamazepine levels had been in the therapeutic range, they are now low. The possible cause for the low carbamazepine levels include: Question options: Dwayne hasn't been taking his carbamazepine because it causes insomnia. Carbamazepine auto-induces metabolism, leading to lower levels in spite of good compliance. Dwayne was not originally prescribed the correct amount of carbamazepine. Carbamazepine is probably not the right antiseizure medication for Dwayne.

b

Elderly patients who are started on levothyroxine for thyroid replacement should be monitored for: Question options: Excessive sedation Tachycardia and angina Weight gain Cold intolerance

b

In addition to methimazole, a symptomatic patient with hyperthyroidism may need a prescription for: Question options: A calcium channel blocker A beta blocker Liothyronine An alpha blocker

b

Inadequate vitamin D intake can contribute to the development of osteoporosis by: Question options: Increasing calcitonin production Increasing calcium absorption from the intestine Altering calcium metabolism Stimulating bone formation

b

Kasey fractured his ankle in two places and is asking for medication for his pain. The appropriate first-line medication would be: Question options: Ibuprofen (Advil) Acetaminophen with hydrocodone (Vicodin) Oxycodone (Oxycontin) Oral morphine (Roxanol)

b

Metformin is a primary choice of drug to treat hyperglycemia in type 2 diabetes because it: Question options: Substitutes for insulin usually secreted by the pancreas Decreases glycogenolysis by the liver Increases the release of insulin from beta cells Decreases peripheral glucose utilization

b

Oral emergency contraception (Plan B) is contraindicated in women who: Question options: Had intercourse within the past 72 hours May be pregnant Are taking combined oral contraceptives Are using a diaphragm

b

Potentially fatal granulocytopenia has been associated with treatment of hyperthyroidism with propylthiouracil. Patients should be taught to report: Question options: Tinnitus and decreased salivation Fever and sore throat Hypocalcemia and osteoporosis Laryngeal edema and difficulty swallowing

b

Sulfonylureas may be added to a treatment regimen for type 2 diabetics when lifestyle modifications and metformin are insufficient to achieve target glucose levels. Sulfonylureas have been moved to Step 2 therapy because they: Question options: Increase endogenous insulin secretion Have a significant risk for hypoglycemia Address the insulin resistance found in type 2 diabetics Improve insulin binding to receptors

b

The Pain Management Contract is appropriate for: Question options: Patients with cancer who are taking morphine Patients with chronic pain who will require long-term use of opiates Patients who have a complex drug regimen Patients who see multiple providers for pain control

b

The mechanism of action of oral combined contraceptives that prevents pregnancy is: Question options: Estrogen prevents the luteinizing hormone surge necessary for ovulation. Progestins thicken cervical mucus and slow tubal motility. Estrogen thins the endometrium making implantation difficult. Progestin suppresses follicle stimulating hormone release.

b

When starting a patient on levothyroxine for hypothyroidism the patient will need follow-up measurement of thyroid function in: Question options: 2 weeks 4 weeks 2 months 6 months

b

Which of the following adverse effects are less likely in a beta1-selective blocker? Question options: Dysrhythmias Impaired insulin release Reflex orthostatic changes Decreased triglycerides and cholesterol

b

Xi, a 54-year-old female, has a history of migraines that do not respond well to OTC migraine medication. She is asking to try Maxalt (rizatriptan) because it works well for her friend. Appropriate decision making would be: Question options: Prescribe the Maxalt, but only give her four tablets with no refills to monitor the use. Prescribe Maxalt and arrange to have her observed in the clinic or urgent care with the first dose. Explain that rizatriptan is not used for postmenopausal migraines and recommend Fiorinal (aspirin and butalbital). Prescribe sumatriptan (Imitrex) with the explanation that it is the most effective triptan.

b

Drug induced constipation classes

benzodiazepines, phenothiazines, seratonin, opiods

Bismuth turns tounge and stools

black/grey

Never used BLANK in drug induced constipation

bulk forming laxative

Most common treatment of constipation in IBS

bulk forming laxatives (psylliom, methylcellulose and calcium polycarbophil), osmotic laxative (Magnesium citrate, milk of mag)

A 56-year-old woman is complaining of vaginal dryness and dyspareunia. To treat her symptoms with the lowest adverse effects she should be prescribed: Question options: Low-dose oral estrogen A low-dose estrogen/progesterone combination A vaginal estradiol ring Vaginal progesterone cream

c

An appropriate first-line drug to try for mild to moderate generalized anxiety disorder would be: Question options: Alprazolam (Xanax) Diazepam (Valium) Buspirone (Buspar) Amitriptyline (Elavil)

c

Ashley comes to the clinic with a request for oral contraceptives. She has successfully used oral contraceptives before and has recently started dating a new boyfriend so would like to restart contraception. She denies recent intercourse and has a negative urine pregnancy test in the clinic. An appropriate plan of care would be: Question options: Recommend she return to the clinic at the start of her next menses to get a Depo Provera shot. Prescribe oral combined contraceptives and recommend she start them at the beginning of her next period and use a back-up method for the first 7 days. Prescribe oral contraceptives and have her start them the same day as the visit with a back-up method used for the first 7 days. Discuss the advantages of using the topical birth control patch and recommend she consider using the patch.

c

Hypoglycemia can result from the action of either insulin or an oral hypoglycemic. Signs and symptoms of hypoglycemia include: Question options: "Fruity" breath odor and rapid respiration Diarrhea, abdominal pain, weight loss, and hypertension Dizziness, confusion, diaphoresis, and tachycardia Easy bruising, palpitations, cardiac dysrhythmias, and coma

c

James has been diagnosed with cluster headaches. Appropriate acute therapy would be: Question options: Butalbital and aspirin (Fiorinal) Meperidine IM (Demerol) Oxygen 100% for 15 to 30 minutes Indomethacin (Indocin)

c

Long-term use of androgens requires specific laboratory monitoring of: Question options: Glucose, calcium, testosterone, and thyroid function Calcium, testosterone, PSA, and liver function Calcium, testosterone, PSA, liver function, glucose, and lipids CBC, testosterone, PSA, and thyroid level

c

When blood glucose levels are difficult to control in type 2 diabetes some form of insulin may be added to the treatment regimen to control blood glucose and limit complication risks. Which of the following statements is accurate based on research? Question options: Premixed insulin analogues are better at lowering HbA1C and have less risk for hypoglycemia. Premixed insulin analogues and the newer premixed insulins are associated with more weight gain than the oral antidiabetic agents. Newer premixed insulins are better at lowering HbA1C and postprandial glucose levels than long-acting insulins. Patients who are not controlled on oral agents and have postprandial hyperglycemia can have neutral protamine Hagedorn insulin added at bedtime.

c

When starting a patient with hypothyroidism on thyroid replacement hormones patient education would include: Question options: They should feel symptomatic improvement in 1 to 2 weeks. Drug adverse effects such as lethargy and dry skin may occur. It may take 4 to 8 weeks to get to euthyroid symptomatically and by laboratory testing. Because of its short half-life, levothyroxine doses should not be missed.

c

Women who are prescribed progestin-only contraception need education regarding which common adverse drug effects? Question options: Increased migraine headaches Increased risk of developing blood clots Irregular vaginal bleeding for the first few months Increased risk for hypercalcemia

c

Stimulant laxatives that can be used as prevention OR maintenace

casanthranol or senna

Stimulant laxative drugs

casanthranol or senna 30-90mg

avoid in pregnancy for laxitive

caster oil and lubricants

Beta blockers have favorable effects on survival and disease progression in heart failure. Treatment should be initiated when the: Question options: Symptoms are severe Patient has not responded to other therapies Patient has concurrent hypertension Left ventricular dysfunction is diagnosed

d

Charlie is a 65-year-old male who has been diagnosed with hypertension and benign prostatic hyperplasia. Doxazosin has been chosen to treat his hypertension because it: Question options: Increases peripheral vasoconstriction Decreases detrusor muscle contractility Lowers supine blood pressure more than standing pressure Relaxes smooth muscle in the bladder neck

d

Drugs used to treat diabetic peripheral neuropathy include: Question options: Metoclopramide Cholinergic agonists Cardioselective beta blockers Gabapentin

d

If interventions to resolve the cause of pain (e.g., rest, ice, compression, and elevation) are insufficient, pain medications are given based on the severity of pain. Drugs are given in which order of use? Question options: NSAIDs, opiates, corticosteroids Low-dose opiates, salicylates, increased dose of opiates Opiates, non-opiates, increased dose of non-opiate Non-opiate, increased dose of non-opiate, opiate

d

Jack, age 8, has attention deficit disorder (ADD) and is prescribed methylphenidate (Ritalin). He and his parents should be educated about the side effects of methylphenidate, which are: Question options: Slurred speech and insomnia Bradycardia and confusion Dizziness and orthostatic hypotension Insomnia and decreased appetite

d

Men who use transdermal testosterone gel (AndroGel) should be advised to avoid: Question options: Washing their hands after applying the gel Wearing occlusive clothing while using the gel Exposure to estrogens while using the gel Skin-to-skin contact with pregnant women while using the gel

d

The drugs recommended for older adults with type 2 diabetes include: Question options: Second-generation sulfonylureas Metformin Pioglitazone Third-generation sulfonylureas

d

The goals of therapy when prescribing hormone replacement therapy (HRT) include reducing: Question options: Cardiovascular risk Risk of stroke or other thromboembolic event Breast cancer Vasomotor symptoms

d

Which of the following statements is true about age and pain? Question options: Use of drugs that depend heavily on the renal system for excretion may require dosage adjustments in very young children. Among the NSAIDs, indomethacin is the preferred drug because of lower adverse effects profiles than other NSAIDs. Older adults who have dementia probably do not experience much pain due to loss of pain receptors in the brain. Acetaminophen is especially useful in both children and adults because it has no effect on platelets and has fewer adverse effects than NSAIDs.

d

The drug of choice for Type 2 dibetics is metformin. Metformin:

decrease glycogenolysis by the liver

Onset of casanthranol or senna

delayed onset- 12-24 hours

Watch for hypokalemia with

diarrhea

Octreotide can be used for

diarrhea 4-8 mg 'TID

Cholestyramine off label use

diarrhea 4g four times a day

Antacids decrease absorption of

digoxin, phenytoin, INH, Ketoconazole

Side effects of anticholinergic

dry mouth, urinary retention, visual disturbances, rebound constipation

Treat fecal impaction and pts with neurogenic bowel with

hyperosmolar- acute for lower bowel

Misoprostol is excreted

in urine

Most common treatment of diarrhea

loperamide

Diet for inflammotroy bowel disease

low fiber diet

Dipeptidyl- peptidase- 4 inhibitors (gliptins) act on the incretin system to improve glycemic control. Advantage of these drugs include

low risk for hypoglycemia

Osmotic laxative drug

mag hydroxide, mag citrate, sodium phosphate, polyethylene glycol

H2 blockers have

many drug interactions and increase alcohol levels

Patients taking antacids should be educated regarding these drugs, including letting them know that:

may cause constipation or diarrhea, high in sodium, separate antacids from other meds by 1 hour

Drugs for opiod induced consipation

methyneltrexone, naloxegol, alvimopan, lubiprostone

Chronic treatment lubricant lacative

mineral oil (OK in kids)

Stops when patient fasts

osmotic

Treatment of IBD needs to needs to be used cautiously in

pregnancy

Antacids increase absorptions of

pseudoephedrine, levodopa

Bulk producing laxative drug

psyllium, methycellulose, pollycarbophil

Magnesium based CI should discuss use with a provider if

renal failure is present

Kelly has diarrhea and is wondering if she can take loperamide (Imodium) for the diarrhea. Loperamide:

slows gastric motility and reduces fluid and electrolyte loss form diarrhea


Conjuntos de estudio relacionados

IFT 302 - Foundations of Information and Computer System Security

View Set

Macroeconomics: Money, Banking, and Financial Institutions

View Set

Final Exam Review Pt. 1 Communicable Diseases

View Set

Research Methods I - Practice Exam 1

View Set